Paediatrics Year 5 Flashcards

1
Q

At her 6-hour neonatal check, a newborn infant is noted to have a flat, pink lesion on the glabella. The registrar notices that it turns darker when she cries. What is this lesion likely to be? A. Capillary hemangioma B. Nevus sebaceus C. Salmon patch D. Café-au-lait spot E. Port wine stain

A

Salmon patch – this is a common finding in neonates, often appearing as a flat pink or salmon- colored lesion on the eyelids, the nuchal area and the glabella. It disappears with age, although those located on the nuchal area tend to persist into adulthood. Capillary hemangioma – also known as a strawberry hemangioma are bright red. Starting off as macules that are slightly raised, they tend to grow rapidly during the first year of life. They represent a small aterio-venous malformation and can appear anywhere on the body. Treatment is not necessary as they regress spontaneously after the first year. Nevus sebaceus – (of Jadessohn) are located on the scalp. They are plaques which are hairless, yellow and resemble warts. As they have the potential to become malignant, they should be removed before adolescence. Café-au-lait spot – these are flat lesions like the salmon patch, however, as the name indicated, they are tan or light brown in color. They can occur anywhere on the body, varying in size and shape. They are sharply demarcated from the underlying skin and are often associated with neurocutaneous disorders such as neurofibromatosis. Port wine stain – this needs to be distinguished from the other more benign birthmarks. Like café- au-lait spots, it is associated with a neurological disorder – specifically Sturge-Weber syndrome. The port wine stain is unilateral, consists of dermal capillaries in the distribution of the ophthalmic division of the Trigeminal nerve (CNV1 – eyelids and upper face).

How well did you know this?
1
Not at all
2
3
4
5
Perfectly
2
Q

At his 6-hour neonatal check, an infant born by spontaneous vaginal delivery is noted to have a large swelling on the back of his head. It only involves the scalp and crosses the suture lines. What is this lesion likely to be? A. Cephalohematoma B. Caput succedaneum C. Subcutaneous fat necrosis D. Meningocele E. Craniosynostosis

A

Caput succedaneum – this is another birth injury that is due to bleeding into the scalp of the presenting part of the head. As it involves the scalp overlying the skull bones it does cross suture lines. They resolve rapidly over several days. Cephalohematoma – this is an example of a birth injury i.e. an injury occurring during labor and delivery. This particular example is a subperiosteal bleed, which doesn’t cross suture lines (Figure 1). This is due to the fact that it is limited to the bone. They can increase in size over the first few days of life and are a common cause of neonatal jaundice. They tend to persist for several months. Subcutaneous fat necrosis – this type of injury tends to occur with instrumental delivery such as forceps or Ventouse. They are firm, rubbery nodules, which can be located anywhere on the body, tending to occur on the back and lower extremities. Meningocele – this is a neural tube defect, spina bifida cystica, where the vertebral arches fail to form and a cyst containing CSF and the meninges protrudes. This condition is associated with low folic acid during pregnancy. Craniosynostosis – these are a series of conditions associated with a variety of autosomal dominant and recessive disorders where the suture lines fuse prior to cessation of brain and head growth. This results in a constellation of developmental delay and characteristic cranio-facial morphology. Craniosynostoses usually occur in association with syndromes, such as Apert and Crouzon syndrome.

How well did you know this?
1
Not at all
2
3
4
5
Perfectly
3
Q

A 4 week old baby presents with jaundice since the 3rd day of life. He was born at term and was of normal birth weight. He is breast fed but is not feeding well and is falling off the centiles. He has a history of bruises and mum noticed a change in the color of his stool. His mother mentions she is blood group O. What would be the most diagnostic investigation? A. Coomb’s Test B. Dipstick Urine C. Liver Function Tests D. TBIDA – radionucleotide scan E. Ultrasound Scan

A

Then answer is D – Infants become clinically jaundice when the bilirubin level climbs to 80 to 120μmol/L. There are three main reasons why more than 60% of infants become clinically jaundiced: (1) decreased life-span of fetal red blood cells (60 to 70 days) compared to adult red blood cells (120 days) (2) high red blood cell concentration in the neonate compared to the adult results in the physiological release of hemoglobin from the breakdown of the these cells and (3) hepatic bilirubin metabolism is immature. There are two reasons why neonatal jaundice is worrying: (1) may indicate an underlying pathology (e.g. hemolytic anemia, infection or metabolic disease) and (2) unconjugated hyperbilirubinemia put the infant at risk of kernicterus. Prolonged jaundice is that which extends beyond 3 weeks of age in a neonate, so this baby has prolonged jaundice. This is usually due to an unconjugated hyperbilirubinaemia but can also be due to liver disease – conjugated hyperbilirubinaemia. Pale stools, dark urine, bleeding/bruising and FTT suggest conjugated hyperbilirubinaemia so we need to check for liver disease. LFTs are not actually very useful for this so the best option here is TBIDA which would show liver uptake and competency of the biliary tree.

How well did you know this?
1
Not at all
2
3
4
5
Perfectly
4
Q

A 4 week old baby is brought in by his mother, with worsening jaundice since one week of age. He is exclusively breast-fed and growing well. Which investigation should be carried out first? A. Conjugated and unconjugated bilirubin B. Coomb’s test C. Liver Function Tests D. Septic Screen E. Thyroid stimulating hormone levels

A

The answer is A – conjugated and unconjugated bilirubin. This will distinguish between a hepatic cause (conjugated) vs. a systemic cause (unconjugated) such as an infection or hemolytic process. LFTs are of little diagnostic value in this case.

How well did you know this?
1
Not at all
2
3
4
5
Perfectly
5
Q

A baby is born at term by normal vaginal delivery. While being breast fed, the mother notices at 6 hours of age he is looking jaundiced. He was immediately placed on phototherapy. While receiving treatment for jaundice, which of the following investigations can help identify the most likely cause? A. Blood culture B. Blood film C. Coomb’s test D. G6PD level E. Urinalysis for reducing substances

A

The answer is C – Coomb’s Test. This child is presenting with jaundice at

How well did you know this?
1
Not at all
2
3
4
5
Perfectly
6
Q

The midwife bleeps you as the FY1 doctor on-call, as she is concerned about a jittery baby. The child was born at 36 weeks via emergency Caesarean section and weight 1.8 kg. He appears to be very sensitive to external stimuli when examined and appears dysmorphic with a poorly developed philtrum above his upper lip. What is the most likely diagnosis? A. Cerebral Infection B. Down’s Syndrome C. Foetal Alcohol Syndrome D. Hypocalcaemia E. Neonatal opiate withdrawal

A

The answer is C – Foetal Alcohol Syndrome Common causes of a ‘jittery’ neonate are: 1. Hypoglycaemia 2. Hypocalcaemia 3. Withdrawal following maternal drug and/or alcohol abuse. Of these, only foetal alcohol syndrome is associated with dysmorphism (saddle-shaped nose, maxillary hypoplasia, absent philtrum and thin upper lip). They are also growth restricted and may have cardiac defects and developmental delay.

How well did you know this?
1
Not at all
2
3
4
5
Perfectly
7
Q

A 15 year old boy presents to the endocrine clinic with persistent gynaecomastia. On examination he is on the 99.6th centile for height, has appropriate pubic hair growth and small testes. His parents are also concerned that he has always been below average at school. What is the most likely chromosomal abnormality? A. 45X B. 47XXY C. Fragile X D. Trisomy 13 E. Trisomy 18

A

The answer is B – 47XXY 47XXY (Klienfelter’s Syndrome) is characterised by micro-orchidism (small testes), tall stature, behavioural problems, gynaecomastia and speech delay. 45X = Turners is a female phenotype. Trisomy 13 (Patau’s) & 18 (Edward’s) = most die

How well did you know this?
1
Not at all
2
3
4
5
Perfectly
8
Q

You are the SHO on call. A two year old boy is brought in. He has been vomiting since yesterday. On examination he has dry mucous membranes; he is tachycardic with sunken eyes, with a reduced skin turgor and prolonged capillary refill time and is lethargic. You give him 20 ml/kg Bolus of 0.9% saline and he improves slightly. Assuming no further vomiting, what is his 24 hour fluid requirement if he weighs 12 kg? A. 240 ml B. 1100 ml C. 1200 ml D. 2060 ml E. 2300 ml

A

The answer is D – 2060ml Calculating fluid requirement: Total fluid requirement is maintenance + deficit + ongoing losses Fluid deficit in ml = % dehydration x weight (kg) x 10 Maintenance fluid = 1st 10kg: 100ml/kg 2nd 10kg: 50ml/kg >20kg: 20ml/kg First, assess the level of dehydration which in this case is severe (10%). Moderate is classified as 5% dehydration where the parameters above are not as severe and the child may be irritable but not lethargic. You can now work out the deficit: 10% x 12kg x 10 = 1200 The maintenance is: (100ml x 10kg) + (50ml x 2kg) = 1100 Total: 1200 + 1100 = 2300ml/24hrs Then minus the 20ml/kg you have already given = 240ml ….. 2300-240 = 2060mls

How well did you know this?
1
Not at all
2
3
4
5
Perfectly
9
Q

A 14 year old boy previously growing normally along the 2nd centile for height is now below the 0.4th centile. He has been complaining of peri-umbilical abdominal pain for the last 6 months on and off. What would be your next step? A. Full Blood Count (FBC) B. Growth Hormone Provocation Test C. Mid-parental height calculation D. Sweat test E. X-ray wrist and hands for bone age

A

The answer is C – mid-parental height calculation 1st step is to assess whether child is growing within his normal range: measure height and weight. Weight is not an option in this choice of answers so sensible to focus on height. The nest best step would be to work out MPH (see red book for how to do this). Then see if 2nd or 0.4th centile lie within the MPH range. This will help to work out whether he is reaching his full potential or not. If he isn’t we can then think about more investigations.

How well did you know this?
1
Not at all
2
3
4
5
Perfectly
10
Q

An Asian mother has brought her 8 week old son to you as her GP, for a routine child health check. He was born at 36 weeks gestation. He is exclusively breast fed and is thriving. Of the following options, which oral supplements would you advise for the mother? A. Calcium B. Folic Acid C. Iron D. Vitamin B12 E. Vitamin D

A

The answer is E – Vitamin D Without any further information apart from ethnicity, mum is most at risk of vitamin D deficiency due to darker skin pigmentation. The amount of vitamin D in breast milk is directly related to maternal levels, therefore increasing the risk of vitamin D deficiency in the baby if mum is not supplemented. (Babies of this age would rarely be exposed to levels of sunlight in Britain adequate enough to synthesise their own vitamin D).

How well did you know this?
1
Not at all
2
3
4
5
Perfectly
11
Q

A 15-year-old boy attends an outpatient paediatric clinic complaining of short stature. He has noticed over the past two years that he has become increasingly shorter than his peers. On examination he looks well. He has pre-pubertal sized testes, and sparse pubic hair. His height has fallen from the 25th to 9th centile. His father reports he was ‘a late developer’. What is the most likely cause of this boy’s short stature? A. Acquired hypothyroidism B. Coeliac Disease C. Constitutional Delay of Growth and Puberty D. Cystic Fibrosis E. Gonadotrophin Deficiency

A

Correct Answer: B: Constitutional Delay of Growth and Puberty In this case, there is evidence of delayed puberty, with consequent delayed pubertal growth spurt. Constitutional delay of growth and puberty is the commonest cause of delayed puberty, and there is often history of delayed puberty in the boy’s father. There was no evidence of systemic disease to suggest either celiac disease or cystic fibrosis as a cause. Isolated gonadotrophin deficiency is rare, occurring in 1 in 10,000 males, and is therefore unlikely.

How well did you know this?
1
Not at all
2
3
4
5
Perfectly
12
Q

A 6 six year old boy presented to the developmental clinic with difficulties fitting into school. Mother states that he has trouble sustaining prolonged conversation. He appears clumsy and has occasional tantrums but he knows the capital of every European country. Of the following options, which member of the multidisciplinary team is LEAST likely to be involved in his care? A. Community Paediatrician B. Neurologist C. Occupational Therapist D. Psychologist E. Special educational needs coordinator

A

Correct Answer: B: Neurologist Asperger’s syndrome is characterised by: 1. Normal intelligence 2. No delay in language development 3.Impaired social and communication skills and a narrow range of obsessional interests Many members of the multidisciplinary team are involved to help manage this disorder and improve a child’s functioning. Doctors involved primarily include neurodevelopmental and community paediatricians and GPs. There is no organic neurological problem in Asperger’s syndrome; therefore a neurologist is least likely to be involved in this child’s care.

How well did you know this?
1
Not at all
2
3
4
5
Perfectly
13
Q

You are the paediatric registrar working in a general out-patient clinic. You see a four year old boy whose parents are concerned that he has difficulty interacting with the neighbours’ children. The teachers in school reprimand him for being cheeky as he often repeats what they say. You notice that he has poor eye contact. What is the next most appropriate step in the management of this patient? A. Initiate Ritalin Treatment B. Reassure symptoms will resolve of their own accord C. Refer to Child Development Team for assessment D. Send the patient to child psychiatry team for assessment E. Send the patient to paediatric neurology

A

Correct Answer: C: Refer to Child Development Team for assessment. The child being described has features of autistic spectrum disorder. The 3 core features of autism which should be present by 3 years are: 1. Impaired social interaction 2. Impaired communication 3. Restricted, repetitive and stereotyped behaviour and interests. Management involves a wide variety of health professionals and varies greatly between different children; therefore a thorough assessment is required to assess each child’s needs. This is done by the child development team.

How well did you know this?
1
Not at all
2
3
4
5
Perfectly
14
Q

A 4-year-old girl is brought to the accident and emergency department after falling off her bed. Radiographic examination reveals with a right-humeral fracture. She has a past medical history of 3 previous attendances for fractures. On examination there is discolouration of the sclera and bowing of the lower limbs. Of the following, which is the most likely diagnosis? A. Achondroplasia B. NAI C. Osteogenesis imperfecta D. Osteoporosis E. Rickets

A

Osteogenesis imperfecta – is an autosomal recessive disorder of collagen synthesis. It typically involves blue sclera, and bowing of the legs, it would also fit to have a past medical history of fractures. Achondroplasia – is an autosomal dominant disorder which is characterized by short stature, large head and prominent forehead with a flattened nasal bridge. The limbs are short, especially the proximal aspects. Patients often have bow-legs and may present with hydrocephalus due to narrowing of the foramen magnum. These children have normal intelligence. Osteoporosis generally doesn’t affect children. Rickets – this occurs in association with vitamin D deficiency

How well did you know this?
1
Not at all
2
3
4
5
Perfectly
15
Q

A 5 year old boy presents with a 12 hour history of a painful left hip on movement. He is afebrile. An x-ray is normal. Ultrasound shows joint effusion in the affected hip. FBC and CXR are normal. What is the best management plan? A. Bed rest B. Joint Aspiration C. Plaster cast D. Skin traction E. Surgical intervention with pin fixation

A

This is transient synovitis (age range is 2-12 yr, child would be afebrile, sudden onset) for which the best treatment option is bed rest, skin traction is used but very rarely. Neutrophils and Acute Phase Reactants are also normal as are blood cultures and x-rays. Together the picture of Transient Synovitis is the opposite of septic arthritis.

How well did you know this?
1
Not at all
2
3
4
5
Perfectly
16
Q

A 5 year old boy is brought into A+E by his annoyed mother. She reveals that he has been constantly complaining of leg pain. On examination you notice a limp and tenderness in his right leg, but no loss of range of movement. You also notice marked bruising on his back. What is the next step in this child’s management? A. Analgesia B. Blood Film C. FBC and Clotting Studies D. Refer to child protection services E. X-ray

A

Correct Answer: X-ray Certain features in the history are suggestive of non-accidental injury (NAI): ‘annoyed mother, sense of delay in presentation to hospital, bruising on back. Full skeletal X-ray is useful in looking for any fractures and this is the best NEXT step. Advice from a senior colleague is always part of the management in a case like this, where NAI might be a possibility.

How well did you know this?
1
Not at all
2
3
4
5
Perfectly
17
Q

A new mother is attending her GP practice for a 6 week check-up for her daughter. On examination the GP notices asymmetrical skin creases around the hip joint which was also unstable. Which is the next most appropriate step for management? A. Arrange for X-ray of the hip B. Ask the Health Visitor to monitor C. Recommend the baby is put in double nappies D. Refer to orthopaedics E. Refer to physiotherapy

A

Correct Answer: D: Refer to orthopaedics Diagnosis is most likely to be Developmental Dysplasia of the Hips (DDH). DDH typically presents with asymmetrical skin-folds, limb abduction, shortening of affected limb and limp (if not diagnosed until child is walking). DDH needs early treatment from Orthopaedics with a Craig’s splint (hips held in abduction) or a Pavlik harness (restraining device) for several months. Therefore physiotherapy, health visitor follow-up and double nappies are inappropriate steps. IN suspected DDH the investigation of choice is an ultrasound scan of the hips, rather than an x- ray.

How well did you know this?
1
Not at all
2
3
4
5
Perfectly
18
Q

You are asked to see a 5-year-old boy who has been carried into the accident and emergency department by his father. His father reports that he has had a bout of the ‘flu’ over the past week and didn’t want to get out of bed this morning. There is no history of trauma. On examination, the boy is afebrile and appears well. He refuses to weight bear because it ‘hurts to walk’. What is the most likely diagnosis? A. Fracture B. Osteomyelitis C. Septic arthritis D. Slipped upper femoral epiphysis E. Transient synovitis

A

The answer is E – this child is refusing to weight bear following a viral illness.

How well did you know this?
1
Not at all
2
3
4
5
Perfectly
19
Q

A 7-year-old child is brought to the clinic by her parents to examine several bumps they have found on her body. Her parents noticed them several weeks ago on her neck and there are now several on her trunk. They say that the child has a tendency to scratch them. On examination – there are several pale-pink papules with umbilicated centers. The child is otherwise fit and well. To which family does the most likely causative viral agent of these lesions belong? A. Herpesviruses B. Poxviruses C. Flaviviruses D. Paramyxoviruses E. Retroviruses

A

Poxviruses – the causative agent is Molluscum contagiosum virus, which is a member of the poxvirus family. The vignette describes the classical presentation. The papules may be slightly pruritic and children tend to scratch and spread them via auto-inoculation (Koebner phenomenon). The papules eventually spontaneously resolve and no treatment is necessary. Herpesviruses – eight types of herpes virus infect humans; HSV1 and HSV2, VZV, EBV, CMV, HHV6 and HHV7 (roseola infantum / subitum – morbilliform rash appearing following fever on chest and abdomen) and HHV8 (Kaposi’s sarcoma-associated). Flaviviruses – Dengue fever and Hepatitis C virus (among others) Paramyxoviruses – Measles (rubeola – highly contagious; 7 to 14 day incubation period; fever, cough, coryza, conjunctivitis, enanthem (Koplik spots) on buccal or labial mucosa, maculopapular rash that spreads cephalocaudally). Live attenuated vaccine given (MMR) at: 12 to 15 months and a second dose age 4 to 6 years. Retroviruses – HIV virus

How well did you know this?
1
Not at all
2
3
4
5
Perfectly
20
Q

A 3-year-old child is brought into the emergency department. His parents are worried as he is running a fever of 38°C and has developed a rash. Examination reveals a rash at several stages of healing – some are clear 4mm vesicles on an erythematous base, while others are crusted. Which of the following complications would you worry about in an immunocompromised child? A. Secondary infection with group A streptococci B. Secondary infection with Pseudomonas aeruginosa C. Hepatic spread D. Pneumonitis E. Encephalitis

A

Pneumonitis – the child in the vignette has Varicella zoster virus infection (chicken pox), which is spread by respiratory droplets. It lies latent in the dorsal root ganglia and leads to herpes zoster with reactivation (shingles). Fifteen to twenty percent of immunocompromised children develop Varicella zoster pneumonitis. Secondary infection with group-A streptococci – this is the most common complication in the normal host causing scarring. Secondary infection with Pseudomonas aeruginosa – this infection commonly occurs in burns victims Hepatic spread – not a complication of Varicella zoster infection Meningitis – while Guillain-Barré syndrome, encephalitis and cerebellar ataxia all may occur in Varicella zoster, meningitis is an uncommon complication.

How well did you know this?
1
Not at all
2
3
4
5
Perfectly
21
Q

You are called to see a 13-month-old boy in the emergency department who over the past two days refuses to weight bear on his right side. The mother says that a week ago he had an ear infection for which the GP prescribed antibiotics. She decided not to give them as he seemed better. Which single investigation would most likely confirm your diagnosis? A. Radiograph of the right hip B. MRI of the right hip C. Blood culture D. Erythrocyte Sedimentation Rate (ESR) E. Full blood count with white blood cell differential

A

Blood culture – this child most likely has osteomyelitis secondary to untreated otitis media. Osteomyelitis can occur due to: (1) hematogenous spread – which is the most common mechanism in children (2) contiguous spread and (3) direct infection from a penetrating wound. Blood cultures are positive in 60% of cases. The most common cause in children aged 3 to 12 years is Staphylococcus aureus. The patient should be treated with IV antibiotics (flucloxacillin and benzyl-penicillin) for 3 weeks in hospital and then switched to oral for 1 to 3 more weeks. Radiograph of the right hip – radiographic findings appear after 10 days to two weeks. They usually show swelling of the surrounding tissue and periosteal elevation. MRI of the right hip – this is used when the diagnosis is uncertain. Erythrocyte Sedimentation Rate (ESR) – may be normal during the early stages of infection Full blood count with white blood cell differential – may be normal during the early stages of infection

How well did you know this?
1
Not at all
2
3
4
5
Perfectly
22
Q

You are the pediatric registrar on call in the emergency department. An 11-month-old child is brought in by her mother with irritability and several episodes of vomiting. The mother says that the child has been like this since waking and has a tactile fever. The mother adds that she is bothered by bright lights. Following a thorough history and examination, you perform a lumbar puncture. Which CSF findings would be in keeping with aseptic meningitis? A. Low glucose B. Leukocytes C. High protein D. High opening pressure E. PMNs

A

Leukocytes – this is the only finding listed that is a typical CSF finding in aseptic meningitis. The rest are all features of a pyogenic infection. Aseptic meningitis is typically caused by a virus, usually an enterovirus. Other viral causes include herpes viruses, adenoviruses, and paramyxoviruses. Enterovirus-associated meningitis is often seen in the summer and fall. Treatment is supportative unless herpes virus – acyclovir. The most common causes of meningitis based on age-group: Neonate to 3/12: GBS Listeria monocytogenes and Escherichia coli Over 3/12: Neisseria meningitides, Streptococcus pneumoniae and Haemophilus influenzae Children younger than 1/12 should receive ampicillin and an aminoglycoside (gentamicin to cover for Listeria) or a cephalosporin (cefotaxime). Children over 1 month should receive ceftriaxone or cefotaxime and ampicillin If Streptococcus pneumoniae is suspected, then vancomycin should be added

How well did you know this?
1
Not at all
2
3
4
5
Perfectly
23
Q

A four year old ‘precious child’ of an ‘alternative’ Chelsea mum presents to A+E with a maculopapular rash which began on the face and is not itchy. On examination the child complains of headache and has prominent suboccipital and post auricular lymphadenopathy. Mother is 7 weeks pregnant with her second child. What is the most appropriate management? A. Advise that the child does not attend nursery for 15 days B. Check maternal serology C. Give mother prophylaxis measles immunoglobulin D. Perform lumbar puncture E. Prescribe oral penicillin

A

Correct Answer: B: Check Maternal Serology The question implies that the child may not have received the MMR vaccine, and therefore the clinical history suggests measles infection. Risk of transmission to mother and foetus is high so mother’s immunity must be checked via maternal serology tests. Rubella infection during pregnancy causes foetal abnormalities such as deafness, cardiac disease, eye problems and mental retardation. The risk is higher in early pregnancy (90% at 9/40) Rubella is a live vaccine so would need to give mother maternal Ig if serology tests show she is not immune. Antibiotics would be of little use in this case as measles is a viral infection.

How well did you know this?
1
Not at all
2
3
4
5
Perfectly
24
Q

An 8 year old boy presented to the GP with a fever lasting 3 weeks, for which he received no medication. It was associated with a cough, loss of appetite and lesions on his lower limbs that were well defined and tender. What is the most likely cause of these symptoms? A. Drug Reaction B. Inflammatory Bowel Disease C. Mycoplasma pneumonia D. Streptococcal infection E. Tuberculosis

A

Correct Answer: E: Tuberculosis The leg lesions are erythema nodosum which can be associated with all of the above. The other symptoms are classically TB, particularly the anorexia and duration of symptoms. There is no indication the child is on any medication, making a drug reaction unlikely. There is also no history of diarrhoea, which would make the diagnosis of inflammatory bowel disease more likely. Streptococcal infections typically are not associated with a cough.

How well did you know this?
1
Not at all
2
3
4
5
Perfectly
25
Q

A mother comes into A+E with her one year old baby who has had a 4 day history of fever, irritability and a rash. The mother says that whilst stroking her child, the skin peels off. On examination purulent crusting around the eyes is noted. What is the most likely diagnosis? A. Kawasaki’s Disease B. Necrotising Cellulitis C. Pemphigoid D. Scalded Skin Syndrome E. Scarlet Fever

A

Correct Answer: D: Scalded Skin Syndrome Staphylococcal scalded skin syndrome (SSSS) presents with a red rash followed by diffuse separation of the epidermal skin. It is associated with fever and irritability and purulent, crusting, localised infection around the eyes, nose and mouth. Desquamation of the fingers and toes can be seen in both Kawasaki’s and scarlet fever, but it is not widespread. The key feature of necrotising cellulitis is necrosis which is not seen here. Pemphigoid is characterised by bullae, which although can be a feature of SSSS, are not mentioned in this case.

How well did you know this?
1
Not at all
2
3
4
5
Perfectly
26
Q

A one year old boy is brought to A+E by his parents with a history of 6 days of high fever. On clinical examination the child is irritable, has cervical lymphadenopathy, red swollen palms, red eyes and a rash. What would be the most appropriate treatment? A. Ibuprofen B. IVIG C. IV penicillin D. Steroids E. Supportive Treatment

A

Correct Answer: C: IVIG This is describing Kawasaki’s disease. The diagnosis is clinical: fever >5 days + 4 other features of: • conjunctival injection • red mucous membranes • cervical lymphadenopathy • rash • red and oedematous palms and soles or peeling of fingers and toes. Treatment is: • IVIG (to reduce risk of coronary artery aneurysms) • Aspirin (to reduce risk of thrombosis, but aspirin is not an option in this question) Scarlet fever (the main differential diagnosis) is treated with penicillin.

How well did you know this?
1
Not at all
2
3
4
5
Perfectly
27
Q

A 2 year old boy presents to Paediatric Accident and Emergency with a four day history of fever, malaise and lymphadenopathy. He saw his GP two days ago and was prescribed amoxicillin. He has since developed a widespread maculopapular rash covering 70% of his body. What is the most likely pathogen causing this presentation? A. Cytomegalovirus B. Epstein-Barr Virus C. Measles D. Human Herpes Virus-6 E. Rubella

A

Correct Answer: B: Epstein-Barr Virus Although all of the above can cause fever, Epstein-Barr Virus is the correct answer. When prescribed amoxicillin or ampicillin, children with EBV infection will sometimes develop a morbilliform rash, not too dissimilar to that seen in measles. The constellation of fever, malaise, lymphadenopathy in the light of this amoxicillin-associated rash should point towards a diagnosis of EBV. Always remember Kawasaki’s Disease should be considered in the differential diagnosis.

How well did you know this?
1
Not at all
2
3
4
5
Perfectly
28
Q

A 4-year-old girl presents to the accident-and-emergency department with a five day history of high fever and a rash. On examination, he is miserable, with a non-purulent conjunctivitis, an erythematous patchy rash and marked cervical lymphadenopathy. What is the most appropriate treatment for this girl’s condition? A. Intravenous ceftriaxone B. Intravenous dexamethasone C. Intravenous diphenhydramine D. Intravenous flucloxacillin E. Intravenous immunoglobulin

A

The answer is E – this child has Kawasaki’s disease.

How well did you know this?
1
Not at all
2
3
4
5
Perfectly
29
Q

A 10 month old boy presents with a two day history of worsening difficulty in breathing and wheeze. Prior to this he was experiencing coryzal symptoms. His 4 year old sister has recently had an upper respiratory tract infection after attending nursery. On examination there is a respiratory rate of 58, nasal flaring, tracheal tug and subcostal and intercostal recession. There is a widespread polyphonic wheeze on auscultation. Pulse oximetry reveals an oxygen saturation of 91% in room air. What is the most likely diagnosis? A. Aspiration B. Bronchiolitis C. Foreign Body inhalation D. Pneumonia E. Pneumothorax

A

Correct Answer: B: Bronchiolitis This is unlikely to be pneumonia as might expect focal signs such as an area of consolidation. A pneumothorax is unlikely as there is no evidence of hyper-resonant percussion notes, decreased breath sounds or asymmetrical chest expansion. With foreign body inhalation, one would expect a more sudden onset of symptoms. There is no history to suggest aspiration as cause of this respiratory distress. A widespread polyphonic wheeze and increased work of breathing, with a history of coryzal illness and close contacts with viral URTIs is a typical presentation of acute viral bronchiolitis.

How well did you know this?
1
Not at all
2
3
4
5
Perfectly
30
Q

A 10 month old boy presents to accident and emergency with a three day history of worsening difficulty in breathing. The boys parents report a fever of 38.5oC measured at home during this time. On examination the child is tachypnoeic and there is slightly reduced chest expansion on the right. On auscultation there is reduced air entry at the right lower zone, and also some crepitations at the right base. You also notice the child is grunting during expiration. Pulse oximetry reveals an oxygen saturation of 91% in room air. What is the most likely diagnosis? A. Aspiration B. Bronchiolitis C. Foreign Body inhalation D. Pneumonia E. Pneumothorax

A

Correct Answer: D: Pneumonia This is most likely to be pneumonia as there are focal signs suggestive of pneumonia. Also end expiratory grunting is often indicative of pneumonia or a lower respiratory tract infection. The grunting is due to glottal closure, which produces a positive end expiratory pressure and helps keep the lower airways patent. Grunting is also seen in infants who are acidotic.

How well did you know this?
1
Not at all
2
3
4
5
Perfectly
31
Q

The nurses have noted that a 2-day-old neonate is turning blue. He was born at term via C- Section. When you examine the baby, he is pale with grey lips, nasal flaring and intercostal recession. The infant is placed on 100% oxygen, but his saturation remains at 82% after 10 minutes. Of the following treatment options what is the most important next step? A. IV ceftriaxone B. IV prostaglandin C. Nebulised Salbutamol D. Oral dexamethasone E. Surfactant therapy

A

IV prostaglandin – failure to improve oxygen saturation in 100% oxygen suggests a cardiac rather than a respiratory problem. The neonate in the vignette most likely has cyanotic congenital heart disease. These infants need immediate treatment with IV prostaglandin to keep the ductus arteriosus patent until definitive treatment can be instituted. Some examples of cyanotic heart disease that present in the neonatal period are: transposition of the great vessels, hypoplastic left heart syndrome and truncus arteriosus. The other treatments are more likely to be useful in treating respiratory disorders (surfactant therapy is specifically useful for hyaline membrane disease).

How well did you know this?
1
Not at all
2
3
4
5
Perfectly
32
Q

During the first day baby check the SHO notices that the baby looks blue. There is nothing on examination of note. No murmur. ABG reveals PaO2 of 2. ECG is normal. Echo shows abnormal arterial connections. What is the best overall management? A. Blalock-Taussig Shunt B. Oxygen and morphine C. Prostaglandin infusion D. Prostaglandin infusion and surgical correction E. Surgery at 6-9 months

A

Correct Answer: D: Prostaglandin infusion and surgical correction Cyanotic CHD with abnormal arterial connections is most likely to be transposition of the great arteries. The treatment for this is to improve the mixing of saturated and desaturated blood and to maintain the patency of ductus arteriosus. This is done by giving a Prostaglandin infusion. Surgical arterial switch procedure is then performed in the first few days of life.

How well did you know this?
1
Not at all
2
3
4
5
Perfectly
33
Q

A 12 hour old 36 week infant, with an uncomplicated pregnancy is on the postnatal ward. It is suddenly noted by the mother that the baby is blue. On examination the infant is centrally cyanosed with a respiratory rate of 70 and marked subcostal and intercostal recession. There is no audible murmur, but a single second heart sound is heard. What is the most likely diagnosis? A. Meconium Aspiration B. Respiratory Distress Syndrome C. Tetralogy of Fallot D. Transient Tachypnoea of the Newborn E. Transposition of the Great Arteries

A

Correct Answer: E: Transposition of the Great Arteries Respiratory distress is seen in neonates less than 34 weeks gestation. Respiratory Distress Syndrome, Transient Tachypnoea of the Newborn and meconium aspiration would most likely present immediately or soon after birth, and not with a sudden onset of cyanosis at 12hrs. Tetralogy of Fallot classically presents with a loud murmur and not cyanosis in early life. Transposition of the Great Arteries has a duct-dependent circulation and usually presents on the 1st/2nd day of life with cyanosis due to ductal closure.

How well did you know this?
1
Not at all
2
3
4
5
Perfectly
34
Q

A 2 year old Caucasian boy presents with a six month history of poor growth and irritability along with foul-smelling, non-bloody diarrhoea. He has had no recent travel history and sweat test was negative. Select the most appropriate step in management from the following options. A. Commence antibiotic therapy B. Commence gluten-free diet C. Steroid therapy D. Substitute cow’s milk for soya milk E. Sulfasalazine

A

Correct Answer: B: Commence gluten-free diet Coeliac disease classically presents with poor weight gain in the 1st 2 years of life, foul smelling chronic diarrhoea, anorexia and abdominal distension. It is managed with lifelong gluten-free diet. Nothing in the history indicates either an infective cause/need for antibiotics, or a relationship to milk intake. Sulfasalazine can be used to treat inflammatory bowel disease.

How well did you know this?
1
Not at all
2
3
4
5
Perfectly
35
Q

A 12 year old girl presents to out-patients with a 6 month history of epigastric pain, 3 inches above the umbilicus, which wakes her up at night. She recently moved to a new secondary school as a result of her parents’ divorce. She has missed 6 weeks of school due to the pain. What is the most appropriate next step in the management of this child? A. Discharge with prophylactic pizotifen B. Reassure that the pain is non-organic and discharge C. Reassure that the pain is non-organic and review with an attendance chart for school D. Trial of treatment with Clarithromycin + Metronidazole + Omeprazole E. USS Kidneys

A

Correct Answer: D: Trial of treatment with Clarithromycin + Metronidazole + Omeprazole Remember Apley’s criteria: If recurrent abdominal pain is peri-umbilical, lasts no more than a few hours, doesn’t wake the child at night and has not been present for a considerable period of time, no investigation is necessary and a non-organic cause is likely. This child has epigastric pain that wakes her at night pointing to possible peptic ulcer disease; therefore a trial of treatment is the most appropriate plan. Other features in the history that suggest a diagnosis of peptic ulcer disease are the two major stressors: moving school and her parent’s divorce. The pain does not sound renal in origin therefore renal imaging is not indicated. Pizotifen can be used to treat abdominal migraine.

How well did you know this?
1
Not at all
2
3
4
5
Perfectly
36
Q

A four month old baby girl presents with a 3 day history of vomiting, fever and poor feeding. This is the second such episode in the last month. Examination is unremarkable with no focal signs except prolonged capillary refill time: RR 35; HR 150. What is the most likely diagnosis? A. Meningitis B. Otitis media C. Pneumonia D. Septicaemia E. UTI

A

Answer: E: UTI UTIs are common in female infants, and lead to dehydration, vomiting, fever and poor feeding. The recurrence is also suggestive of a UTI. Other options would have had more clinical signs: Meningitis – photophobia, neck stiffness, maybe a non-blanching rash Pneumonia – RR is very important as part of the assessment and isn’t even mentioned in the question OM – ear symptoms, maybe diarrhoea Septicaemia – systemic rash, more severe shock

How well did you know this?
1
Not at all
2
3
4
5
Perfectly
37
Q

Which of the following would be diagnostic of a UTI in a 5 year old presenting with fever, rigors and dysuria? A. Bacterial culture of >105 cfu of E.coli from absorbent pads in a nappy B. Bacterial culture of >106 cfu of mixed organisms from a clean catch C. Bacterial culture of >103 cfu of Pseudomonas from a clean catch D. Bacterial culture of >104 cfu of Proteus sp from a suprapubic aspirate E. Proteinuria +++ on urine dipstick from clean catch

A

Answer: D: Bacterial culture of >104 cfu of Proteus sp from a suprapubic aspirate Answer D is correct as a suprapubic aspirate sample is far less likely to be contaminated than either a clean catch or a sample obtained from an absorbable nappy. Proteinuria is a non-specific sign. Nitrites on urine dipstick would be more indicative of UTI.

How well did you know this?
1
Not at all
2
3
4
5
Perfectly
38
Q

A five year old boy was admitted to the paediatric ward after presenting to A+E with a two day history of sore knees and a purpuric rash over the buttocks and back of his legs. A full blood count is normal. Of the following options, what is the most appropriate next investigation? A. Blood clotting studies B. Blood cultures C. Joint aspirate of knees D. Lumbar puncture E. Urine dipstick

A

Correct Answer: E: Urine Dipstick The most likely diagnosis in this case is Henoch-Schönlein Purpura (HSP). Henoch-Schönlein purpura is an immune-mediated vasculitis. It can include: - Palpable purpura on buttocks & extensor surfaces of limbs - Arthralgia & periarticular oedema - Abdominal pain - Glomerulonephritis (leading to) haematuria >80% of children with HSP have micro- or macroscopic haematuria or mild proteinuria. This is the best next step as it is a simple non-invasive bedside test.

How well did you know this?
1
Not at all
2
3
4
5
Perfectly
39
Q

You are asked to see 3 year old boy who has bilateral leg, ankle and scrotal oedema. You also notice some facial oedema, which the parents report has subsided slightly since the morning. What is the next investigation you will perform? A. Anti-streptolysin O Titre B. Hepatitis B Serology C. Full Blood Count D. Urea and Electrolytes E. Urine Dipstick

A

Correct Answer: E: Urine Dipstick Urine dipstick is the only test from these options you can perform at the bedside and is therefore an appropriate next step, as it will be of diagnostic benefit in what sounds like the nephrotic syndrome. ASOT would be a useful test as post-streptococcal glomerulonephritis may be a cause of the nephrotic syndrome. This arises as immune-complexes containing streptococcal antigens are deposited in the glomerular basement membrane, resulting in an acute proliferative glomerulonephritis. Test for Hepatitis B antigen and antibodies may be of use as HBV-associated glomerulonephritis can also be a cause of the nephrotic syndrome. This occurs as the result of cryoglobulinaemia sometimes associated with HBV. These cryoglobulins precipitate in the glomerular basement membrane, resulting in a membranous glomerulonephritis.

How well did you know this?
1
Not at all
2
3
4
5
Perfectly
40
Q

A 7-year-old boy attends the GP with his parents. He has just started a new school and his parents report that he has started wetting the bed. A urine dipstick is negative or nitrites and leukocytes. How should the GP proceed at this point? A. Prescribe Amitriptyline B. Prescribe Desmopressin C. Suggest an enuresis alarm D. Explore school-related issues E. Prescribe Fluoxetine

A

The answer is D – this child has secondary enuresis most likely as a result of a school-related problem. The GP needs to delineate exactly what is happening before proceeding to suggest medical and non-medical management

How well did you know this?
1
Not at all
2
3
4
5
Perfectly
41
Q

A 14-year-old girl presents to the GP with her mother. She gives a five-week-history of recurrent headache associated with nausea, vomiting and abdominal pain. She is on no regular medications. Her mother also had similar episodes at her age. What is the most likely diagnosis? A. Benign Intracranial Hypertension B. Migraine C. Cluster headache D. Posterior Fossa Tumor E. Tension Headache

A

Migraine – is typically associated with nausea, vomiting, and abdominal pain. There is often a positive family history in migraine. Benign intracranial hypertension – typically affects obese young women, with a peak incidence in the 3rd or 4th decades of life. The intensity of the headache is positional – it is usually worse on waking, and relieved by standing up. Cluster headaches – these typically occur in young adult men. They tend to occur at a similar time each day, lasting several days to weeks at a time before disappearing. Posterior fossa tumor – the most common intra-cranial tumors in children are located in the posterior fossa. They typically present with a history of worsening headache over weeks to months, and may also display features secondary to cranial nerve or cerebellar compression such as strabismus (CN VI), nystagmus or ataxia. Tension headaches – these often present with a characteristic band-like distribution across the forehead or behind the eyes.

How well did you know this?
1
Not at all
2
3
4
5
Perfectly
42
Q

A worried mother brings her 9-year-old son to the GP. He suffered from a discrete episode of visual changes, staggering and falling yesterday. He subsequently developed a headache. This is the third time it has happened in three months. He is afebrile and cranial nerve examination is normal at the GP. What is the most likely diagnosis? A. Atonic epileptic seizure B. Benign paroxysmal vertigo C. Migraine D. Reflex anoxic seizures E. Syncope

A

Correct Answer: C: Migraine Child is too old for R.A.S – this is usually in infants or toddlers and follows a trigger such as cold food, head trauma, a sudden fright or a fever. BPV is associated with nystagmus and viral labyrinthitis, neither of which are mentioned. Epileptic seizure would have more details of post-ictal state and temperature would be higher. There is no mentioned trigger for syncope Best option is migraine for which this child is in the typical age range, there are also visual and abdominal symptoms and recurrence – characteristic of a migraine.

How well did you know this?
1
Not at all
2
3
4
5
Perfectly
43
Q

A 15 year old girl with a BMI of 29 presents with a one month history of frontal headache which is unresponsive to analgesia. The pain worsens on coughing and sneezing and is associated with nausea. A thorough history reveals she is struggling at school and has a family history of glaucoma. The most likely diagnosis is? A. Benign intracranial hypertension B. Migraine without aura C. Refractive error D. Sinusitis E. Space-occupying lesion

A

BIH shows the signs of an ICP but is ‘benign’ in that there is no space-occupying lesion; it is a diagnosis of exclusion and is more common in over-weight females.

How well did you know this?
1
Not at all
2
3
4
5
Perfectly
44
Q

Sophie, a 6 year old known epileptic, comes to A+E having been seizing for the last 35 minutes. The on-call registrar has administered one dose of rectal diazepam. However, Sophie is still seizing. IV access has been gained. Select the next most appropriate treatment. A. Lorazepam (iv) B. Midazolam (oral) C. Paraldehyde (pr) D. Phenobarbital (infusion) E. Phenytoin (iv)

A

Correct Answer: A: Lorazepam Once intravenous access has been gained, Lorazepam is preferential to rectal diazepam or buccal midazolam. Paraldehyde, phenytoin and phenobarbital are considered next if the child is not responding to benzodiazepines.

How well did you know this?
1
Not at all
2
3
4
5
Perfectly
45
Q

You are the on-call pediatric registrar in the accident-and-emergency department. A mother brings her 6-year-old son in with severe shortness-of-breath. There is some improvement in his status following administration of nebulized salbutamol and oxygen. However, the child is a known asthmatic and you feel he requires admission. The mother doesn’t wish to stay in hospital and leaves the department with the child. What is the appropriate next step? A. Contact the consultant on-call B. Call hospital security C. Call the police D. Call social services E. Respect the mother’s decision and document this in the notes

A

Except for choice E (respect the mother’s decision and document this in the notes) all of the above choices should be instituted. However, as the most appropriate next step is C – call the police. This is due to the fact that the mother has left the premises and the child is still at significant risk of harm. If the mother were threatening to leave the hospital, it would be appropriate to call hospital security.

How well did you know this?
1
Not at all
2
3
4
5
Perfectly
46
Q

A mother brings her 5-month-old infant into the accident-and-emergency department with coryzal symptoms. His weight and length are less than the fifth centile. His birth weight was 4.1kg and he is 3.6kg when the nurses weight him today. He takes an unknown amount of cows-milk-based formula per day and porridge. What is the most likely cause of his weight loss? A. Cows-milk-protein allergy B. Non-organic failure to thrive C. Celiac disease D. Inflammatory bowel disease E. Weight loss secondary to reflux

A

Non-organic failure to thrive – this is often seen when a child isn’t fed a sufficient amount of calories to sustain growth and development. This may occur when there is insufficient parental knowledge about feeding, substance abuse, poverty or underlying social and emotional disturbance. Presentation is variable – the child is often thin, with prominence of the bones, wasting of the buttocks (much like celiac disease) and is often developmentally delayed. There may also be evidence of physical abuse, such as a torn frenulum, burns or unusual bruises. Social services should be informed to assess the home situation and it is often necessary to hospitalize the child for nutritional assessment and unlimited feedings.

How well did you know this?
1
Not at all
2
3
4
5
Perfectly
47
Q

Which of these Western European countries has the highest incidence of teenage pregnancies? France Portugal Norway United Kingdom Holland

A

UK Becoming a teenage mother may be a positive life-choice, especially if there is considerable support from extended family. However, when the pregnancy is unintended, there can be many adverse consequences for mother and child, especially if she is unsupported or living in poverty.

How well did you know this?
1
Not at all
2
3
4
5
Perfectly
48
Q

Claire is 14 years old. She visits her general practitioner as she wants to go on the oral contraceptive pill. She is sexually active and has a boyfriend with whom she has been in a relationship for 6 months. Her mother is not aware she has come to see the doctor today and is also not aware she is sexually active. Claire does not want her mother knowing she is getting the oral contraceptive pill. She has no other medical problems. She has regular periods and her blood pressure is normal. What is the recommended advice to give her? You will only give her the pill if her parents are present to consent in writing You will prescribe the pill and promise not to tell her parents You will prescribe the pill and encourage her to tell her mother she is going to start the pill You will prescribe the pill but will inform her parents you are doing so She cannot have the pill as she is not legally allowed to have sex

A

You will prescribe the pill and encourage her to tell her mother she is going to start the pill It is usually desirable for the parents to be informed and involved in contraception management. She should be encouraged to tell them or allow the doctor to, but if the young person is competent to make these decisions for herself, in the UK the courts have supported medical management of these situations without parental knowledge.

How well did you know this?
1
Not at all
2
3
4
5
Perfectly
49
Q

Which of the following is the most common cause of death in adolescence in the UK? Cancer Heart disease Infection Injury and poisoning Neurological disease

A

Injury and Poisoning In the UK injury and poisoning account for 60% of deaths in 15–19-year-olds. Alcohol is thought to be a contributing factor in a third of those deaths.

How well did you know this?
1
Not at all
2
3
4
5
Perfectly
50
Q

Olivia is a 16-year-old girl who presents to the local pharmacy for ‘emergency contraception’. She has had unprotected sex with her boyfriend, who is also 16 years old. She has no other medical problems and is not currently on any medications. How long after unprotected intercourse can emergency contraception be used? 12 hours 24 hours 48 hours 72 hours 1 week

A

72 hours Emergency contraception is available from a pharmacist without prescription for those aged 16 years and over, and on prescription for those under 16 years. If taken within 72 hours, it has a 2% failure rate.

How well did you know this?
1
Not at all
2
3
4
5
Perfectly
51
Q

Sam, 16 years old, is brought to his general practitioner because his parents are worried about him. They complain that he stays locked in his room and is not doing well at school. He often refuses to get out of bed because he has a headache. You speak to Sam alone. He complains of being bored all the time, and says that there is nothing he enjoys. He feels hopeless. He no longer goes out with his friends. He complains of a headache, which is of gradual onset and like a tight band around his head. He is a fan of ‘ER’, a medical drama, where one of the characters suffered from a brain tumour. He is convinced that he also has a brain tumour. He has no other medical problems. He takes paracetamol for his headache, with good effect. Acne vulgaris Asthma Depression Epilepsy Chronic Fatigue Syndrome Anorexia Nervosa Somatic symptoms Risk-taking behaviour Pregnancy Malignancy Psychosis

A

Depression

How well did you know this?
1
Not at all
2
3
4
5
Perfectly
52
Q

Gareth is a 15-year-old boy who has become increasingly withdrawn. He frequently argues with his mother, particularly when she believes he should stay at home to do his homework rather than leaving the house to meet his new group of friends. His mother believes that this new group of friends are using drugs. She is very upset that he has even been cautioned by the police, for urinating in public whilst intoxicated. He has no other medical problems. Acne vulgaris Asthma Depression Epilepsy Chronic Fatigue Syndrome Anorexia Nervosa Somatic symptoms Risk-taking behaviour Pregnancy Malignancy Psychosis

A

Risk-taking behaviour Gareth is a 15-year-old boy who has become increasingly withdrawn. He frequently argues with his mother, particularly when she believes he should stay at home to do his homework rather than leaving the house to meet his new group of friends. His mother believes that this new group of friends are using drugs. She is very upset that he has even been cautioned by the police, for urinating in public whilst intoxicated. He has no other medical problems.

How well did you know this?
1
Not at all
2
3
4
5
Perfectly
53
Q

Bennu, an Egyptian 14-year-old girl, is sent to the school nurse by her maths teacher because she is complaining of a headache. She complains of a headache at least once a week, and often feels tired. There is no pattern to her headaches. She has no other symptoms and is doing well at school. She has lots of friends and is a sociable teenager. There are no abnormalities on examination. She has no other medical problems and is on no medications. Acne vulgaris Asthma Depression Epilepsy Chronic Fatigue Syndrome Anorexia Nervosa Somatic symptoms Risk-taking behaviour Pregnancy Malignancy Psychosis

A

Somatic symptoms Bennu, an Egyptian 14-year-old girl, is sent to the school nurse by her maths teacher because she is complaining of a headache. She complains of a headache at least once a week, and often feels tired. There is no pattern to her headaches. She has no other symptoms and is doing well at school. She has lots of friends and is a sociable teenager. There are no abnormalities on examination. She has no other medical problems and is on no medications.

How well did you know this?
1
Not at all
2
3
4
5
Perfectly
54
Q

Anika, a 15-year-old girl, presents with her mother to her general practitioner. She complains of vomiting. This is present most commonly in the morning and is associated with abdominal pain, fatigue and breast tenderness. You ask her if she has had unprotected sex, and she denies this. She has a suprapubic mass. She has no other medical problems and is not on any medications. Acne vulgaris Asthma Depression Epilepsy Chronic Fatigue Syndrome Anorexia Nervosa Somatic symptoms Risk-taking behaviour Pregnancy Malignancy Psychosis

A

Pregnancy Anika, a 15-year-old girl, presents with her mother to her general practitioner. She complains of vomiting. This is present most commonly in the morning and is associated with abdominal pain, fatigue and breast tenderness. You ask her if she has had unprotected sex, and she denies this. She has a suprapubic mass. She has no other medical problems and is not on any medications.

How well did you know this?
1
Not at all
2
3
4
5
Perfectly
55
Q

Keith, a 15-year-old boy, attends paediatric outpatient department with his mother. Over the last 6 weeks he has become increasingly aggressive, and last week was suspended from school for throwing his book at a teacher. His mother says that this behaviour is completely out of character; he was previously a kind and hard-working boy. His school performance has also deteriorated. He also complains of an occipital headache that is worse in the morning or when he bends down. His mother sometimes finds vomit in the sink. Examination is normal except for a squint. On checking his eye movements, you find that he is unable to deviate his left eye laterally. He has no other medical problems. He has tried taking paracetamol for his headache but this has not been therapeutic. Acne vulgaris Asthma Depression Epilepsy Chronic Fatigue Syndrome Anorexia Nervosa Somatic symptoms Risk-taking behaviour Pregnancy Malignancy Psychosis

A

Malignancy Keith, a 15-year-old boy, attends paediatric outpatient department with his mother. Over the last 6 weeks he has become increasingly aggressive, and last week was suspended from school for throwing his book at a teacher. His mother says that this behaviour is completely out of character; he was previously a kind and hard-working boy. His school performance has also deteriorated. He also complains of an occipital headache that is worse in the morning or when he bends down. His mother sometimes finds vomit in the sink. Examination is normal except for a squint. On checking his eye movements, you find that he is unable to deviate his left eye laterally. He has no other medical problems. He has tried taking paracetamol for his headache but this has not been therapeutic.

How well did you know this?
1
Not at all
2
3
4
5
Perfectly
56
Q

Worldwide, which of the following causes the most deaths in children under 5 years old? Neonatal problems Diarrhoea Malaria Pneumonia Injuries

A

Neonatal problems

How well did you know this?
1
Not at all
2
3
4
5
Perfectly
57
Q

Ritha, aged 3 months, is admitted to hospital with a 2-day history of mild coryza, and difficulty breathing. She has been feeding poorly for the last 3 weeks. Which clinical feature most supports her having congenital heart disease, rather than respiratory disease? Sibling with congenital heart disease Poor feeding Generalised wheeze on auscultation Marked hepatomegaly Ejection systolic murmur, grade II/VI at left sternal edge

A

Marked hepatomegaly. In infants, this is an important sign of heart failure, usually secondary to congenital heart disease

How well did you know this?
1
Not at all
2
3
4
5
Perfectly
58
Q

Nazma, aged 4 years, presents with a 1-week history of several episodes of central abdominal pain. She is of Indian ethnicity, but the family live in Kenya and are visiting relatives in the UK. She is otherwise well. Her relative’s general practitioner thinks she may be slightly pale and that her spleen is enlarged, as it is 3 cms below the costal margin. There are no other abnormalities on examination. Which of the following is the most likely cause for her enlarged spleen? Acute lymphoblastic leukaemia (ALL) Malaria Hookworm infestation Wilms tumour Sickle cell disease

A

Malaria. High prevalence in Kenya and may cause chronic anaemia and splenomegaly (Note - not sure if I agree with this one, as malaria would more likely present with intermittent fevers and being unwell….? However, this is the answer given in the book!)

How well did you know this?
1
Not at all
2
3
4
5
Perfectly
59
Q

Katie, an 18-month-old Caucasian girl, is reviewed in the paediatric clinic. She is unsteady on her feet. She walks with a limp and tends to fall to her left side. Her limb tone and reflexes are as follows: - Right arm - tone and reflexes normal - Left arm - tone increased, reflexes brisk - Right leg - tone and reflexes normal - Left leg - tone increased, reflexes brisk Which is the site of her neurological lesion? Upper motor neurone lesion Lower motor neurone lesion Cerebellar lesion Basal ganglia lesion Neuromuscular junction

A

The increased tone and reflexes of her left arm and leg are from an upper motor neurone lesion, most likely in the right side of her brain.

How well did you know this?
1
Not at all
2
3
4
5
Perfectly
60
Q

Katie, an 18-month-old Caucasian girl, is reviewed in the paediatric clinic. She is unsteady on her feet. She walks with a limp and tends to fall to her left side. Her limb tone and reflexes are as follows: - Right arm - tone and reflexes normal - Left arm - tone increased, reflexes brisk - Right leg - tone and reflexes normal - Left leg - tone increased, reflexes brisk Which of the following best describes the pattern of neurological signs? Diplegia Right hemiplegia Left hemiplegia Spastic quadriplegia Choreoathetoid cerebral palsy

A

Katie has a left hemiplegia due to the increased tone and reflexes on her arm and leg.

How well did you know this?
1
Not at all
2
3
4
5
Perfectly
61
Q

Jeremiah, a 6-year-old boy, is brought by his mother to the ophthalmology outpatient clinic. She is worried about her son’s ‘funny eyes’. You examine him using the cover test, and you find the following: - Both eyes open - right eye deviates inwards - Left eye covered - right eye fixes on stimulus What disorder does he have? Right convergent squint Right divergent squint Alternating convergent squint Left convergent squint Left divergent squint

A

Right convergent squint It is a convergent squint because the eye is facing inwards towards the midline. It affects the right eye when both eyes are uncovered. The uncovered left eye usually focuses on objects. But when the left eye is covered the right eye takes up a normal position and focuses.

How well did you know this?
1
Not at all
2
3
4
5
Perfectly
62
Q

William, a 9-year-old boy from Slovakia, presents to the rapid access paediatric clinic with a fractured tibia following a fall from a wall. He is otherwise well but has a history of shortness of breath and wheeze when running. His mother denies that he has ever needed treatment for his wheeze. He is not unwell currently. Which clinical sign is caused by long-term, poorly treated obstructive lung disease, and is seen as indentation at the lower ribcage at the costal margin? Barrel chest Pectus excavatum Pectus carinatum Harrison sulcus Sternal recession

A

Harrison sulcus

How well did you know this?
1
Not at all
2
3
4
5
Perfectly
63
Q

Ishmael, a 15-year-old boy from Pakistan, is seen in the outpatient department. He has a long history of chest infections needing recurrent courses of antibiotics. He has a productive cough. He opens his bowels once a day. On examination, he has a normal temperature, his skin and mucous membranes are pink and his heart sounds are normal. His hands look unusual. On auscultation, there are some scattered crepitations at both bases. In view of his recurrent chest infections, you had ordered a sweat test, which is negative. Explain his clubbing? Cystic fibrosis Infective endocarditis Bronchiectasis Crohn disease Tetralogy of Fallot

A

This child has marked clubbing of the fingers due to bronchiectasis. Although cystic fibrosis is the commonest cause, it may also be caused by other conditions.

How well did you know this?
1
Not at all
2
3
4
5
Perfectly
64
Q

Rob, a 9-month-old boy, presents with fever and difficulty breathing for the last 3 days. His breathing is now interfering with his feeding. Examination of his chest finds: - subcostal and intercostal recession - RR = 70/min - Scattered wheezes - Hyperinflated chest - Fine end-expiratory crackles - HR = 120/min - HS 1 + 2 + 0 Give the most likely diagnosis: Acute exacerbation of asthma Bronchiolitis Chronic asthma Cystic fibrosis Heart failure Inhaled foreign body (left side) Inhaled foreign body (right side) Pleural effusion (left-sided) Pleural effusion (right-sided) Pneumonia (left-sided) Pneumonia (right-sided) Pneumothorax (left-sided) Pneumothorax (right-sided)

A

Bronchiolitis

How well did you know this?
1
Not at all
2
3
4
5
Perfectly
65
Q

Hatem, a 3-year-old boy, presents with fever and difficulty breathing, getting worse for the last 3 days. Examination shows: - RR = 40 - Dullness on percussion in lower right zone, as well as bronchial breathing and inspiratory crepitations - HR = 120/min - HS 1 + 2 + 0 Give the most likely diagnosis: Acute exacerbation of asthma Bronchiolitis Chronic asthma Cystic fibrosis Heart failure Inhaled foreign body (left side) Inhaled foreign body (right side) Pleural effusion (left-sided) Pleural effusion (right-sided) Pneumonia (left-sided) Pneumonia (right-sided) Pneumothorax (left-sided) Pneumothorax (right-sided)

A

Pneumonia (right-sided)

How well did you know this?
1
Not at all
2
3
4
5
Perfectly
66
Q

Darren, a 3-year-old boy, was eating peanuts 2 days ago when his younger brother pushed him over. He coughed up the peanuts and was all right. Today, he has been coughing and becomes breathless as soon as he runs about. He is afebrile. Examination findings are: - RR = 36/min - Apex beat displaced to the right - Reduced air entry on the left - Percussion is normal Give the most likely diagnosis: Acute exacerbation of asthma Bronchiolitis Chronic asthma Cystic fibrosis Heart failure Inhaled foreign body (left side) Inhaled foreign body (right side) Pleural effusion (left-sided) Pleural effusion (right-sided) Pneumonia (left-sided) Pneumonia (right-sided) Pneumothorax (left-sided) Pneumothorax (right-sided)

A

Inhaled foreign-body (left side)

How well did you know this?
1
Not at all
2
3
4
5
Perfectly
67
Q

Tony, a 4-year-old boy, is admitted with pneumonia. His chest-ray shows consolidation at the right base. In spite of antibiotic therapy, he remains febrile and unwell. Examination findings are: - RR = 50 - Lower right zone is dull to opercussion, absent breath sounds, scattered inspiratory crepitations Give the most likely diagnosis: Acute exacerbation of asthma Bronchiolitis Chronic asthma Cystic fibrosis Heart failure Inhaled foreign body (left side) Inhaled foreign body (right side) Pleural effusion (left-sided) Pleural effusion (right-sided) Pneumonia (left-sided) Pneumonia (right-sided) Pneumothorax (left-sided) Pneumothorax (right-sided)

A

Pleural effusion (right-sided)

How well did you know this?
1
Not at all
2
3
4
5
Perfectly
68
Q

Jamalah, a 7-year-old girl, presents with difficulty breathing. She has had a cold for the last 2 days. This is the third time this has happened, each time when she had a cold. She has not had any other medical problems. On examination, she has an upper respiratory infection and the signs listed here: - rr = 44/min - Mild subcostal and intercostal recession - Hyper-resonance on percussion - scattered wheezes Give the most likely diagnosis: Acute exacerbation of asthma Bronchiolitis Chronic asthma Cystic fibrosis Heart failure Inhaled foreign body (left side) Inhaled foreign body (right side) Pleural effusion (left-sided) Pleural effusion (right-sided) Pneumonia (left-sided) Pneumonia (right-sided) Pneumothorax (left-sided) Pneumothorax (right-sided)

A

Acute exacerbation of asthma

How well did you know this?
1
Not at all
2
3
4
5
Perfectly
69
Q

Steven has just had his first birthday party. During his party he commando-crawled with great speed, although he cannot walk. He managed to pick off all the Smarties (round chocolate sweets) from his birthday cake. He can say two words with meaning. After his birthday party, he impressed his guests by waving goodbye. Which area of Steven’s development is delayed? Gross motor Fine motor and vision Speech and hearing Social, emotional and behavioural development None of the above

A

He has therefore achieved normal milestones for a 12-month-old Note also that commando crawlers and bottom-shufflers tend to decide upon walking a little later than normal crawlers

How well did you know this?
1
Not at all
2
3
4
5
Perfectly
70
Q

Gerald is a 16-month-old boy who has not yet said his first word, and does not babble much. His mother believes he does not hear well because he doesn’t startle when a door slams or show any response to his name. His development is otherwise normal. Which test would be best to assess Gerald’s hearing? Evoked otoacoustic emission Auditory brainstem response audiometry Distraction hearing test Visual reinforcement audiometry Speech discrimination testing

A

Visual reinforcement audiometry This is the most reliable test for a child Gerald’s age. The test requires an assistant to play with the child and keep their attention. Behind a soundproof window, another assistant will play sounds through a loudspeaker at particular frequencies. When the child turns around to the noise, a glass-fronted box with a toy inside that was previously dark lights up as visual reinforcement to reward the child for turning round

How well did you know this?
1
Not at all
2
3
4
5
Perfectly
71
Q

Evie is 10 days old and was born in London. Her health visitor reviews her and her parents at home. She is feeding well and has a normal examination except that she has a squint. The health visitor mentions to parents she will keep this under review. At what age does Evie need to be referred to an ophthalmologist if the squint is still present? 2 weeks 6 weeks 12 weeks 8 months 12 months

A

12 weeks A newborn may appear to squint when looking at nearby objects because their eyes over-converge. By 6 weeks, the eyes should move together when following an object, and by 12 weeks there should be no squint present

How well did you know this?
1
Not at all
2
3
4
5
Perfectly
72
Q

Sophie is a well 8-week-old baby who was born at term. She has come for a routine developmental check. Which of the following would you not expect her to be able to do? Raise her head when lying prone Smile Fix and follow a face or interesting object Reach out and grasp an object Quieten to a loud noise

A

Reach out and grasp an object An 8-week-old infant will be able to do the remaining 4 activities, but still only has a primitive grasp reflex, so will not be able to voluntarily grasp an object. She will only be able to grasp what is placed in her hand

How well did you know this?
1
Not at all
2
3
4
5
Perfectly
73
Q

Joanna is an active toddler. She is just being potty-trained, and has had several days where she has remained dry. She enjoys pulling her clothes off to use the potty, but cannot dress herself again. She enjoys playing by pretending to make her mother a cup of tea, but does not play well with her older siblings, as she has not yet learnt how to take turns. She is very bossy, and demands things by saying ‘give me’ or ‘me drink’. She can build a tower of six blocks, and enjoys running and climbing on furniture. What developmental age is Joanna? 12 months 18 months 24 months 2 ½ years 3 years

A

24 months Joanna is dry by day, can undress and has symbolic play. She is not yet playing interactively; she will learn this at about 3 years of age. She is constructing two-word sentences. She constructs a tower of six blocks and can run.

How well did you know this?
1
Not at all
2
3
4
5
Perfectly
74
Q

Cordelia is a 4-month-old baby who is assessed by her general practitioner because of constant crying and poor feeding. She is fed by bottle on infant formula. Her mother tearfully complains that she is finding it very difficult to cope. She also has a 20-month-old boy who has recently been referred to the speech and language therapist because of language delay. She used to work as a solicitor, and her partner is a company director who often travels abroad. Charlotte’s developmental, growth and physical examination is normal. What is the likely cause of Cordelia’s problems? An inherited genetic condition Gastro-oesophageal reflux Maternal postnatal depression and stress Pyloric stenosis Down syndrome

A

Maternal postnatal depression and stress This may be detrimental to Cordelia’s development, as infants are totally dependent on their main caregiver. Her older brother may have speech delay because he is not receiving enough stimulation to develop his own language skills or there may be an underlying problem causing his speech delay.

How well did you know this?
1
Not at all
2
3
4
5
Perfectly
75
Q

At what age will children achieve the milestones described in the scenarios (median age)? Each age can be used once, more than once, or not at all. Rosette can build a three-cube tower and can point to her nose. 6 weeks 6 months 8 months 10 months 12 months 18 months 2 years 3 years 4 years 5 years

A

12 months

How well did you know this?
1
Not at all
2
3
4
5
Perfectly
76
Q

At what age will children achieve the milestones described in the scenarios (median age)? Each age can be used once, more than once, or not at all. Grace enjoys drawing. She has just learnt to copy drawing a square, and can build steps using blocks after being shown. 6 weeks 6 months 8 months 10 months 12 months 18 months 2 years 3 years 4 years 5 years

A

4 years

How well did you know this?
1
Not at all
2
3
4
5
Perfectly
77
Q

At what age will children achieve the milestones described in the scenarios (median age)? Each age can be used once, more than once, or not at all. Lizzie has just started crawling. 6 weeks 6 months 8 months 10 months 12 months 18 months 2 years 3 years 4 years 5 years

A

8 months

How well did you know this?
1
Not at all
2
3
4
5
Perfectly
78
Q

At what age will children achieve the milestones described in the scenarios (median age)? Each age can be used once, more than once, or not at all. Ivan can tie his shoe laces all by himself. 6 weeks 6 months 8 months 10 months 12 months 18 months 2 years 3 years 4 years 5 years

A

5 years

How well did you know this?
1
Not at all
2
3
4
5
Perfectly
79
Q

At what age will children achieve the milestones described in the scenarios (median age)? Each age can be used once, more than once, or not at all. Herbert can transfer objects from one hand to the other whilst sitting without support and with a straight back. 6 weeks 6 months 8 months 10 months 12 months 18 months 2 years 3 years 4 years 5 years

A

8 months

How well did you know this?
1
Not at all
2
3
4
5
Perfectly
80
Q

At what age will children achieve the milestones described in the scenarios (median age)? Each age can be used once, more than once, or not at all. Blessing’s mother is so pleased; her baby has just learnt to smile when she smiles at her! 6 weeks 6 months 8 months 10 months 12 months 18 months 2 years 3 years 4 years 5 years

A

6 weeks

How well did you know this?
1
Not at all
2
3
4
5
Perfectly
81
Q

At what age will children achieve the milestones described in the scenarios (median age)? Each age can be used once, more than once, or not at all. Rita’s father is thrilled that she has just said her first word – says ‘dada’ to her father only. 6 weeks 6 months 8 months 10 months 12 months 18 months 2 years 3 years 4 years 5 years

A

10 months

How well did you know this?
1
Not at all
2
3
4
5
Perfectly
82
Q

At what age will children achieve the milestones described in the scenarios (median age)? Each age can be used once, more than once, or not at all. Anthony has a friend at nursery, and they enjoy playing with toy cars together. 6 weeks 6 months 8 months 10 months 12 months 18 months 2 years 3 years 4 years 5 years

A

3 years

How well did you know this?
1
Not at all
2
3
4
5
Perfectly
83
Q

At what age will children achieve the milestones described in the scenarios (median age)? Each age can be used once, more than once, or not at all. Peace has just taken her first steps! 6 weeks 6 months 8 months 10 months 12 months 18 months 2 years 3 years 4 years 5 years

A

12 months

How well did you know this?
1
Not at all
2
3
4
5
Perfectly
84
Q

At what age will children achieve the milestones described in the scenarios (median age)? Each age can be used once, more than once, or not at all. Rinah can follow her mother’s two-step commands, such as ‘fetch your red shoes from the cupboard’. 6 weeks 6 months 8 months 10 months 12 months 18 months 2 years 3 years 4 years 5 years

A

3 years

How well did you know this?
1
Not at all
2
3
4
5
Perfectly
85
Q

At which of these ages is the following action (screening, examination, health promotion activity) taken in the child health surveillance and promotion programme in the United Kingdom? Each answer can be used once, more than once, or not at all. Hearing test using audiometry Newborn 5-6 days 12 days 8 weeks 3 months 4 months 8 months 12 months 13 months 2-3 years 4-5 years (pre-school) 5 years (school entry)

A

13 months

How well did you know this?
1
Not at all
2
3
4
5
Perfectly
86
Q

At which of these ages is the following action (screening, examination, health promotion activity) taken in the child health surveillance and promotion programme in the United Kingdom? Each answer can be used once, more than once, or not at all. First routine immunisation with 5 in 1 immunisation - DTaP/IPV, Hib, PCV (diphtheria, tetanus, pertussis, polio, Haemophilus influenza, pneumococcal) Newborn 5-6 days 12 days 8 weeks 3 months 4 months 8 months 12 months 13 months 2-3 years 4-5 years (pre-school) 5 years (school entry)

A

8 weeks

How well did you know this?
1
Not at all
2
3
4
5
Perfectly
87
Q

At which of these ages is the following action (screening, examination, health promotion activity) taken in the child health surveillance and promotion programme in the United Kingdom? Each answer can be used once, more than once, or not at all. Guthrie test (biochemical screening test) Newborn 5-6 days 12 days 8 weeks 3 months 4 months 8 months 12 months 13 months 2-3 years 4-5 years (pre-school) 5 years (school entry)

A

5-6 days

How well did you know this?
1
Not at all
2
3
4
5
Perfectly
88
Q

At which of these ages is the following action (screening, examination, health promotion activity) taken in the child health surveillance and promotion programme in the United Kingdom? Each answer can be used once, more than once, or not at all. An orthoptist assessment for visual impairment Newborn 5-6 days 12 days 8 weeks 3 months 4 months 8 months 12 months 13 months 2-3 years 4-5 years (pre-school) 5 years (school entry)

A

4-5 years (preschool)

How well did you know this?
1
Not at all
2
3
4
5
Perfectly
89
Q

At which of these ages is the following action (screening, examination, health promotion activity) taken in the child health surveillance and promotion programme in the United Kingdom? Each answer can be used once, more than once, or not at all. Advice on reducing the risk of sudden infant death syndrome by ‘Back to sleep’, avoiding overheating and avoiding parental smoking Newborn 5-6 days 12 days 8 weeks 3 months 4 months 8 months 12 months 13 months 2-3 years 4-5 years (pre-school) 5 years (school entry)

A

Newborn

How well did you know this?
1
Not at all
2
3
4
5
Perfectly
90
Q

At which of these ages is the following action (screening, examination, health promotion activity) taken in the child health surveillance and promotion programme in the United Kingdom? Each answer can be used once, more than once, or not at all. Hearing test using evoked otoacoustic emission or auditory brainstem response audiometry Newborn 5-6 days 12 days 8 weeks 3 months 4 months 8 months 12 months 13 months 2-3 years 4-5 years (pre-school) 5 years (school entry)

A

Newborn

How well did you know this?
1
Not at all
2
3
4
5
Perfectly
91
Q

At which of these ages is the following action (screening, examination, health promotion activity) taken in the child health surveillance and promotion programme in the United Kingdom? Each answer can be used once, more than once, or not at all. First MMR (measles/mumps/rubella) immunisation Newborn 5-6 days 12 days 8 weeks 3 months 4 months 8 months 12 months 13 months 2-3 years 4-5 years (pre-school) 5 years (school entry)

A

13 months

How well did you know this?
1
Not at all
2
3
4
5
Perfectly
92
Q

Jonathan is 4 years old and lives in a small village in southern England. He attends a paediatric outpatient clinic with his grandmother who is his legal guardian. She is concerned that he only seems to like to play with his toy train and insists on watching the same DVD every night before he goes to bed. He attends nursery where he plays with the toys but not with other children. His behaviour can be very difficult to manage at times. He does not say any words whereas the grandmother’s children were speaking in sentences at his age. On examination you notice he does not make eye contact with you and pushes his toy train back and forth on the floor. The rest of his examination is normal. What is the most likely diagnosis? Attention deficit hyperactivity disorder (ADHD) Autistic spectrum disorder Developmental coordination disorder (DCD, also known as dyspraxia) Asperger syndrome Expressive language disorder

A

Autism spectrum disorder Autism is a triad of impaired social interaction, speech and language disorder and ritualistic and repetitive behaviour.

How well did you know this?
1
Not at all
2
3
4
5
Perfectly
93
Q

At what age does autistic spectrum disorder usually become evident? 0–12 months 12–24 months 2–4 years 4–8 years Above 8 years

A

2-4 years Autistic spectrum disorder usually presents at this age because this is when language and social skills rapidly develop

How well did you know this?
1
Not at all
2
3
4
5
Perfectly
94
Q

At what age would you expect the clinical features of cerebral palsy to become evident? 0–12 months 12–24 months 2–4 years 4–8 years Above 8 years

A

0-12 months Cerebral palsy most often presents during this time when acquisition of motor skills occurs most rapidly.

How well did you know this?
1
Not at all
2
3
4
5
Perfectly
95
Q

Fortunate is an 8-month-old black African girl who was born at term. She is seen in the paediatric outpatient department. She can roll over. She does not crawl. She can say ‘dada’, but says it to everyone not just her father. She reaches out and grasps objects with her left hand but not with her right, and puts objects in her mouth. She smiles, but is not able to wave bye-bye. Select the option that concerns you most about Fortunate’s development. Unable to crawl Left hand preference Unable to use sounds discriminately to parents Unable to wave bye-bye None of the above

A

Left hand preference Fortunate has developed a preference for using her left hand at 8 months. Development of a hand preference before 1 year of age is abnormal, and she needs to be assessed for hemiplegic cerebral palsy affecting her right side.

How well did you know this?
1
Not at all
2
3
4
5
Perfectly
96
Q

Gloria is a 19-month-old white British girl whose health visitor is concerned because she is still only babbling and says no distinct words. She is able to walk, scribbles with crayons and feeds herself with a spoon. What is the most appropriate first action? Hearing test Assessment by a team specialising in autistic spectrum disorders Reassure the health visitor Refer to an ENT surgeon Refer to a paediatrician for a full developmental assessment

A

Hearing test Speech delay can be due to hearing impairment, and this should be assessed first prior to referring her to a specialist.

How well did you know this?
1
Not at all
2
3
4
5
Perfectly
97
Q

Andrew is 5 years old. His father feels his behaviour has deteriorated and he is worried he is not hearing him all the time. He has poor articulation of the few words that he can say. Andrew goes to the audiology department and has his hearing tested. His audiogram shows: What type of hearing loss does he have?

A

Mild conductive hearing loss in the right ear. The audiogram shows mild conductive hearing loss in the right ear. It is mild at 30-50dBHL and conductive as high frequency hearing is relatively preserved and bone conduction (same for both ears) is normal.

How well did you know this?
1
Not at all
2
3
4
5
Perfectly
98
Q

Cruz is a 2-year-old boy. He has recently moved to the UK from Mexico. He attends the audiology department as he has marked problems with his language development. Below is his audiogram:

What type of hearing loss does he have?

Mild conductive hearing loss in both ears
Severe conductive hearing loss in both ears
Mild sensorineural hearing loss in both ears
Severe sensorineural hearing loss in both ears
Mixed hearing loss in both ears

A

Severe sensorineural loss in both ears. The hearing loss is severe (70–90 dBHL) to profound (>90 dBHL) and sensorineural, as there is no significant air-bone gap.

99
Q

Jenny is an 8-week-old girl who was born pre-term at 35 weeks’ gestation. She is seen by her general practitioner for her surveillance review. Her mother is concerned that she does not smile. Her gross motor development appears to be normal and she startles to loud noises. However, she will not follow a face nor a colourful ball. The appearance of one of her eyes is shown here. The other eye has a similar appearance. She has no other medical problems.

What is the likely underlying diagnosis?

Corneal trauma
Cataract
Conjunctivitis
Retinopathy of prematurity
Vitamin A deficiency

A

Cataract

This picture shows a white reflex in the eye that is caused by the clouding of the lens

100
Q

Which of these investigations would you choose to confirm the diagnosis of developmental delay in the children described in the following scenarios? Each investigation can be used once, more than once, or not at all.

Dorcus is a 9-month-old Ugandan infant who attends clinic because of unusual movements. She has developed episodes of suddenly throwing her head and arms forward. These occur in repetitive bursts. She was able to sit and babble but has stopped doing so.

Chromosome karyotype

DNA-FISH analysis

TFTs

Maternal amino acids for rasied phenylalanine

Creatine kinase

Blood lactate

Congenital infection screen

Cranial USS

CT/MRI of the brain

EEG

A

EEG

101
Q

Which of these investigations would you choose to confirm the diagnosis of developmental delay in the children described in the following scenarios? Each investigation can be used once, more than once, or not at all.

Geoffrey is a 10-month-old Afro-Caribbean baby referred to the child development clinic. His mother raised concerns because he is slower in his development than her four other children. He can sit, but only if he is propped up with cushions. He is not crawling or pulling to stand. On examination he is hypotonic, with some dysmorphic features, including upslanting palpebral fissures, there is a skin fold of the upper eyelid covering the inner corner of the eye and a flat occiput. He has no other medical problems except some vitiligo. He was born at term by normal vaginal delivery. He has been slow to feed.

Chromosome karyotype

DNA-FISH analysis

TFTs

Maternal amino acids for rasied phenylalanine

Creatine kinase

Blood lactate

Congenital infection screen

Cranial USS

CT/MRI of the brain

EEG

A

Chromosome karyotype

102
Q

Which of these investigations would you choose to confirm the diagnosis of developmental delay in the children described in the following scenarios? Each investigation can be used once, more than once, or not at all.

Darren is a 3-year-old boy who has difficulty climbing stairs. He always needs to hold on to the railings or to have a supporting hand. He walked unsupported at 14 months. His development is otherwise normal. On examination the power in his legs is reduced, he is somewhat hypotonic but the reflexes are normal.

Chromosome karyotype

DNA-FISH analysis

TFTs

Maternal amino acids for rasied phenylalanine

Creatine kinase

Blood lactate

Congenital infection screen

Cranial USS

CT/MRI of the brain

EEG

A

Creatine Kinase

103
Q

Which of these investigations would you choose to confirm the diagnosis of developmental delay in the children described in the following scenarios? Each investigation can be used once, more than once, or not at all.

Clarissa, a cheerful 20-month-old white British girl, is referred to the child development clinic by her health visitor because she is not yet walking. She was born at term with no complications. She learnt to sit without support at 10 months, and is able to crawl, although she drags her right leg behind her. Her mother says that she has always been left-handed. Examination revealed reduced power and reflexes but increased tone in the right arm.

Chromosome karyotype

DNA-FISH analysis

TFTs

Maternal amino acids for rasied phenylalanine

Creatine kinase

Blood lactate

Congenital infection screen

Cranial USS

CT/MRI of the brain

EEG

A

CT/MRI of the brain

104
Q

Which of these investigations would you choose to confirm the diagnosis of developmental delay in the children described in the following scenarios? Each investigation can be used once, more than once, or not at all.

Batar is a 1-week-old baby born at term who is seen in the ophthalmology clinic because of cataracts. He has a head circumference of 32cm (normal range) and weight of 2.3kg, mild jaundice, pallor and moderate hepatosplenomegaly. His mother thinks that he does not hear well. His mother had a mild flu-like illness during pregnancy.Chromosome karyotype

DNA-FISH analysis

TFTs

Maternal amino acids for rasied phenylalanine

Creatine kinase

Blood lactate

Congenital infection screen

Cranial USS

CT/MRI of the brain

EEG

A

Congenital infection screen

105
Q

Considering these types of cerebral palsy, choose the type of movement disorder to fit the scenarios below. Each type of cerebral palsy can be used once, more than once, or not at all.

Alan is a 3-year-old boy who developed a preference for using his left hand at 7 months of age. He learnt to sit at 9 months and walked at 20 months. When he runs, he holds his right arm flexed and limps with his right foot. On examination, his right upper and lower limbs are stiff, with increased reflexes.

Spastic hemiplegia

Spastic diplegia

Spastic quadriplegia

Dyskinetic

Hypotonic-ataxic

A

Spastic hemiplegia

106
Q

Considering these types of cerebral palsy, choose the type of movement disorder to fit the scenarios below. Each type of cerebral palsy can be used once, more than once, or not at all.

Ronaldo is a 6-year-old boy. He attends the outpatient department as his teacher has had some concerns. He has recently started school and has been noted to be unsteady on his feet. He has to walk and run with his legs quite wide apart to stop himself from falling over. His teacher also reports that he finds it difficult to grip a pen and to write because of unsteadiness.

Spastic hemiplegia

Spastic diplegia

Spastic quadriplegia

Dyskinetic

Hypotonic-ataxic

A

Hypotonic-ataxic

107
Q

Considering these types of cerebral palsy, choose the type of movement disorder to fit the scenarios below. Each type of cerebral palsy can be used once, more than once, or not at all.

Gerald is a 4-year-old boy who was born in Tanzania, and had severe jaundice as a neonate that could not be treated because of lack of medical services. He now has abnormal movements of all his limbs where he adopts and maintains unusual postures, and when he is startled by a loud noise the arm on one side straightens and the opposite arm bends. When he is asleep he is hypotonic.

Spastic hemiplegia

Spastic diplegia

Spastic quadriplegia

Dyskinetic

Hypotonic-ataxic

A

Spastic quadriplegic

108
Q

Considering these types of cerebral palsy, choose the type of movement disorder to fit the scenarios below. Each type of cerebral palsy can be used once, more than once, or not at all.

Hassan is 3 years old and was born at 26 weeks gestation weighing 700g. He sat at 10 months, and has just started to walk. He can scribble and build a tower of 3 blocks. When you examine him, he is walking on tiptoes, and his legs ‘scissor’ when you lift him up. He is able to feed himself.

Spastic hemiplegia

Spastic diplegia

Spastic quadriplegia

Dyskinetic

Hypotonic-ataxic

A

Spastic diplegia

109
Q

Considering these types of cerebral palsy, choose the type of movement disorder to fit the scenarios below. Each type of cerebral palsy can be used once, more than once, or not at all.

Moses is a 5-year-old boy who failed to attain his developmental milestones from shortly after birth. Currently, he cannot roll or talk, but he can smile. His mother complains it is difficult to dress him as both his arms and legs are stiff. On examination, his left and right upper and lower limbs are stiff and hyper-reflexic. He has a primitive grasp reflex in both hands.

Spastic hemiplegia

Spastic diplegia

Spastic quadriplegia

Dyskinetic

Hypotonic-ataxic

A

Spastic quadriplegia

110
Q

Which of the following health professionals involved in the care of a disabled child would most help the children described below?

Gloria is a 22-month-old girl whose health visitor is concerned because she is still only babbling and says no distinct words. She is able to walk, scribbles with crayons and feeds herself with a spoon.

Dietician

Social worker

Psychologist

Paediatrician

Speech and Language

Occupational Therapist

Physiotherapist

Specialist Health Visitor

A

Speech and Language Therapist

111
Q

Which of the following health professionals involved in the care of a disabled child would most help the children described below?

Cathy is 15 years old and was in a road traffic accident. She spent a week in intensive care and needed an operation on her spine. She is currently not able to walk and has been shown how to use a wheelchair. Her parents are desperate to get her back home. They live in a town house where her bedroom is on the first floor, and therefore her father would need to carry her up the stairs.

Dietician

Social worker

Psychologist

Paediatrician

Speech and Language

Occupational Therapist

Physiotherapist

Specialist Health Visitor

A

Occupational Therapist

112
Q

Which of the following health professionals involved in the care of a disabled child would most help the children described below?

Bilal is a 6-year-old boy who has recently been diagnosed with Duchenne muscular dystrophy. This presented with weakness and easy fatiguability when walking. He finds it difficult to co-ordinate his movements. He is currently in mainstream school and needs to be able to move around the classroom.Dietician

Social worker

Psychologist

Paediatrician

Speech and Language

Occupational Therapist

Physiotherapist

Specialist Health Visitor

A

Physiotherapist

113
Q

Which of the following health professionals involved in the care of a disabled child would most help the children described below?

Thomas is a 9-year-old boy who has suspected Asperger syndrome. He has problems interacting with his siblings and classmates. His academic performance at school is poor. He has a very strict daily routine and becomes very upset if this is broken. He sleeps poorly at night.

Social worker

Psychologist

Paediatrician

Speech and Language

Occupational Therapist

Physiotherapist

Specialist Health Visitor

A

Psychologist

114
Q

Which of the following health professionals involved in the care of a disabled child would most help the children described below?

Frankie is a 6-month-old infant from the UK. She has always struggled to gain weight and never took her bottles of milk. When she was fed she would often have choking episodes which led to episodes of pneumonia. She subsequently needed to have a naso-gastric (NG) tube and gastrostomy so she could be fed. Her mother wants to start trying to feed her some solid food.

Social worker

Psychologist

Paediatrician

Speech and Language

Occupational Therapist

Physiotherapist

Specialist Health Visitor

A

Speech and Language Therapist

115
Q

Which of the following health professionals involved in the care of a disabled child would most help the children described below?

Adrianna is a 4-year-old girl who has recently moved from Poland to the UK. She has severe learning difficulties and attends a special nursery. She has epilepsy, which is difficult to control, and is on two different antiepileptic drugs. She continues to have seizures despite this medication. She does not have an underlying diagnosis and does not have co-ordinated care.

Social worker

Psychologist

Paediatrician

Speech and Language

Occupational Therapist

Physiotherapist

Specialist Health Visitor

A

Padiatrician

116
Q

Which of the following health professionals involved in the care of a disabled child would most help the children described below?

Jake is a 20-month-old boy who burnt himself on a radiator whilst playing unsupervised. He attended the Accident and Emergency department where analgesia was given and dressings applied. He was seen by a paediatrician who performed a more detailed examination looking for signs of child abuse. There were no concerns and the child was discharged home to be seen later in the burns clinic.

Social worker

Psychologist

Paediatrician

Speech and Language

Occupational Therapist

Physiotherapist

Specialist Health Visitor

A

Specialist Health Visitor

117
Q

Which of the following health professionals involved in the care of a disabled child would most help the children described below?

Sian is 2 years old. She is being followed-up for failure to thrive (growth faltering). All her investigations have come back normal but she is still not gaining adequate weight. She drinks a lot of dilute squash but her mother complains she won’t eat any of the food she gives her.

Social worker

Psychologist

Paediatrician

Speech and Language

Occupational Therapist

Physiotherapist

Specialist Health Visitor

A

Dietician

118
Q

A 5-year-old girl is brought to A+E after school by her mother and school teacher with central abdominal pain.

The best history is likely to be obtained by talking to:

Her mother

Her mother and her school teacher

Her school teacher

The A+E triage nurse

The girl herself and her mother

A

A- The girl herself and her mother

119
Q

A two-year-old boy is seen in Paediatric Out-Patients with a 6 month history of diarrhoea and parental concern about his nutritional status. The best way to assess these concerns would be to:

Assess his weight for height

Calculate his Body Mass Index (BMI)

Measure his height and weight in clinic

Measure his upper arm circumference

Review and plot serial weights and heights

A

A- Review and plot serial weights and heights

120
Q

A previously healthy 2-year-old child presents with a 24 hour history of diarrhoea and vomiting. Which of the following is the single, most accurate method for assessing the degree of dehydration?

Assess skin turgor

Assess the fontanelle

Calculate the difference between the current weight and the predicted weight from the child’s growth records

Examine the mucous membranes

Measure the heart rate and blood pressure

A

A- Calculate the difference between the current weight and the predicted weight from the child’s growth records

Assessing dehydration in D&V

  • skin turgor - normal would be to spring back instantly - however, in connective tissue disorders, loss of subcutaneous fat etc this test is totally useless.
  • assessing the fontanelle is only useful in the very young
  • examining the mucus membranes - good method.
  • heart rate and blood pressure are affected at later stages e.g. Around 10%
  • difference between the current weight and predicted weight from the child’s growth. There should be enough data from the first year of life to predict an appropriate target weight. This works better in children that previously well and acute dehydrated. Most accurate of the above methods.

For any of the above clinical signs to work (not the growth chart one) need to be at least 3-5% dehydrated.

If 15% dehydrated, look moribund.

121
Q

A 3-year-old boy attends the Paediatric A+E Department because he has developed an itchy rash whilst at a birthday party. Of the following features, which requires immediate treatment with 0.01 ml/kg of 1:1000 adrenaline i.m.?

Blood pressure of 88/50

Generalised urticaria

Lip swelling

Respiratory rate of 22/minute

Wheeze on auscultation

A

Wheeze on auscultation

BP of 88/50 in a 3 year old- normal!

Resps of 22 - also normal!

Wheeze on auscultation with history of allergy - sign of impending anaphylaxis, give adrenalin! 0.01ml/kg of 1:1000 IM.

122
Q

A 3-year-old boy has just started fitting in A+E. You are the F1 doctor. The nurses are all busy with other children. Of the following actions which should you do first?:

Administer rectal diazepam

Gain iv access

Give him high-flow oxygen

Measure his blood sugar

Do a blood gas

A

A- Give him high-flow oxygen

123
Q

One evening a 4-month-old boy is brought into A+E by his step-father because of a prolonged nose bleed. This has now stopped. The examination is normal apart from some irregular bruising on his abdomen and small, well circumscribed marks on his back. The next most appropriate step from the options below would be to:

Admit him and arrange a full skeletal survey, clinical photographs and clotting studies

Check his full blood count and clotting studies, and discharge him if they are normal for follow up in OPD

Discharge him and discuss the case the next day with the child protection team

Discuss the case with the child protection team now

Reassure the step-father that since the nose bleed has stopped, no further treatment is required

A

A- Discuss the case with the child protection team now

124
Q

A 2-month-old, ex-26 week gestation boy establishing feeding on the neonatal unit has developed a distended abdomen and bile-stained vomiting, and has an increasing oxygen requirement. The most likely diagnosis is:

Appendicitis

Gastro-oesophageal reflux

Hirschsprung’s disease

Intussusception

Necrotising enterocolitis

A

A- Necrotising enterocolitis

125
Q

A 5-month-old boy has been brought to A+E 2 hours previously with a 12 hour history of very high fever and vomiting. Examination does not reveal a clear source of infection so a blood culture is taken and lumbar puncture is performed. Of the following options, which method reflects current clinical practice when an urgent urine sample is required prior to starting antibiotics?

Fixing a sterile bag and waiting for a sample

Performing an in-out urinary catheterisation with aseptic technique

Placing a sterile pad in the nappy and waiting for a sample

Placing and leaving a urinary catheter in situ

Waiting for the parents to ‘catch’ a mid-stream urine sample in a bowl

A

A- Performing an in-out urinary catheterisation with aseptic technique

126
Q

A 2-week-old boy is brought to A+E by his mother who has noticed that he has become increasingly jaundiced. She reported that he had not been breast-feeding as well as previously, and also commented that his stools looked like “off-white chewing gum”. The most likely diagnosis is:

Breast-milk jaundice

Congenital gall stones

Extra-hepatic biliary atresia

Glucose 6-phosphate dehydrogenase deficiency

Rhesus incompatibility

A

A- Extra-hepatic biliary atresia

127
Q

A 4-month-old girl is brought into A+E Resus by paramedics with a widespread, progressing, non-blanching purpuric rash on both lower limbs. She has cold peripheries with a capillary refill time of 5 seconds, has a heart rate of 180/min and her blood pressure is unrecordable. The most likely diagnosis is:

Anaphylaxis

Erythema infectiosum

Group B streptococcal infection

Henoch-Schonlein purpura (HSP)

Neisseria meningitidis infection

A

A- Neisseria meningitidis infection

128
Q

A 12-year-old boy has presented to A+E unable to weight-bear on his right leg due to pain in the hip. He grazed his thigh 6 days ago whilst playing football, and for the last 48 hours has had high fevers. Bloods taken today show a CRP of 240 and a WCC of 24.5. A hip x-ray shows a periosteal reaction in the proximal femur but no fracture or joint effusion. The most likely diagnosis is:

Osteomyelitis

Perthe’s disease

Septic arthritis

Slipped upper femoral epiphysis

Transient synovitis

A

A- Osteomyelitis

129
Q

A 2-year-old boy presented to A+E with a 4 week history of intermittent high fever and misery. The parents have noticed a widespread evanescent salmon-pink rash. In the last 48 hours he has been unwilling to weight bear. Laboratory tests reveal markedly raised acute phase markers. The most likely diagnosis is:

Chronic listeria infection

Dermatomyositis

Duchenne muscular dystrophy

Juvenile idiopathic arthritis

Systemic lupus erythematosus

A

A- Juvenile idiopathic arthritis

130
Q

A 14-month-old boy, with normal growth is referred to you due to his mother complaining that he is a difficult feeder. He refuses most solids and meal times are a battle, but he likes drinking cows’ milk and has about 1 litre per day. Which of the following would you do next?

No need to test for deficiency in the absence of clinical signs

Test for iron deficiency

Test for vitamin B12 deficiency

Test for folic acid deficiency

Test for calcium deficiency

A

A- Test for iron deficiency

131
Q

A 15-month-old girl, whose mother wears a burka, is referred to you by the health visitor. She was not meeting her gross motor developmental milestones and is generally very irritable. She used to grow on the 50th centile, but is now on the 25th. She has 5 older siblings and gets 2 hourly breast feeds and consumes a very little amount of solids with no additional dairy products. What is the most likely diagnosis?

Failure to thrive

Iron Deficiency

Phosphate deficiency

Nutritional Rickets

Vitamin A deficiency

A

A- Nutritional Rickets

Breast fed too much without weaning after a year? Vitamin d deficiency leading to nutritional ricketts - their gross motor development is delayed, may present with e.g. fractures. Particularly in vitamin d deficient mothers.

Note that breast feeding for too long will lead to iron deficiency anaemia.

132
Q

A 15 year old girl attends the Emergency Department with her boyfriend, also 15, requesting the morning-after pill following a condom accident the previous evening. She reveals that four months ago she was circumcised during a family trip to Somalia. She understands your advice and the implications of her decisions to engage in sexual activity. She refuses to inform her parents. The couple are using condoms regularly. What is the appropriate management?

Decline to prescribe the morning-after pill and refer the patient back to her GP

Decline to prescribe the morning-after pill and inform her parents so that they can support her

Prescribe the morning-after pill and recommend that the girl informs her parents

Prescribe the morning-after pill and immediately alert the safeguarding children’s team

Prescribe the morning-after pill, give contraceptive advice and inform her parents

A

A- Prescribe the morning-after pill and immediately alert the safeguarding children’s team

133
Q

You are working a night shift on a general paediatric ward. Which of the following presentations would most strongly indicate calling in the emergency respiratory physiotherapist?

Child with cystic fibrosis on a planned admission for i.v. antibiotics

Eight year old with asthma in respiratory distress

Eleven year old boy with Duchenne muscular dystrophy, respiratory tract infection and rising pCO2

One year old with bronchiolitis

Two year old with pneumonia and a dry, non-productive cough

A

A- Eleven year old boy with Duchenne muscular dystrophy, respiratory tract infection and rising pCO2

134
Q

An 8-year-old girl is referred to outpatient department with a 3-month history of acne, breast development and fine pubic hair. She is otherwise healthy and has had no known illnesses, nor is she on any long term medication. Examination is otherwise normal. What is the most likely aetiology?

a feminising ovarian tumour

a gonadotropin producing tumour

congenital adrenal hyperplasia, late onset

Cushing’s disease

Early onset of normal puberty

A

A- Early onset of normal puberty

135
Q

A 13-year-old boy presents to his GP for a routine physical prior to participation in competitive sports. He has had no recent illnesses and no past medical history of note. His height and weight are on the 75th centile for his age. Cardiovascular examination reveals a grade 2/6 ejection systolic murmur heard loudest at the left lower sternal border. It is low pitched and musical but does not radiate. Which of the following is the most likely diagnosis?

atrial septal defect

ventricular septal defect

vibratory innocent murmur

tricuspid regurgitation

mitral stenosis

A

A- vibratory innocent murmur

135
Q

A 10-year-old boy is being seen in the clinic for the evaluation of obesity. His development has been appropriate until this point, and no major illnesses have been reported. His body mass index is 32 kg/m2. Which of the following statements is most accurate about obesity in children?

the incidence is decreasing

associated abnormal physical findings are common

laboratory tests are usually not helpful in confirming the diagnosis

weight reduction programmes are unsafe for prepubertal patients

family history is not important

A

A- laboratory tests are usually not helpful in confirming the diagnosis

136
Q

An 8-week-old exclusively breastfed baby boy was born at 38 weeks gestation after an uneventful pregnancy. Which of the following findings would require a prompt referral to a paediatric surgeon?

bilateral nontender scrotal swellings which transilluminate

a right testis which can be manipulated to the base of the scrotal sac

an acute episode of balanitis

glandular hypospadias

an irreducible firm lump which extends from the inguinal canal to the scrotum

A

A- an irreducible firm lump which extends from the inguinal canal to the scrotum

137
Q

A healthy 4-year-old girl develops acute onset of petechiae and epistaxis. Laboratory findings include haemoglobin = 12g/dL; white cell count = 5.5 with normal differential; and platelet count = 15. Of the following investigations, which would you do next?

blood film

bone marrow aspirate

check the bleeding time

clotting studies

platelet antibody tests

A

A- blood film

ITP will present with a well child (recent history of viral illness) with thrombocytopenia. Do a blood film to rule out malignancies.

138
Q

The laboratory calls you to inform you that a baby who is now 10 days old has an elevated TSH on their Guthrie Card test. If this condition is left untreated, which of the following signs is the baby most likely to demonstrate in the first few months of life?

Hyperphagia

prolonged jaundice

diarrhoea

hyperirritability

hyperreflexia

A

A- prolonged jaundice

139
Q

A 2-week-old boy is admitted with a 3-day history of vomiting and increasing lethargy. Physical examination is normal except for increased pigmentation of the areolar and nipples bilaterally. Laboratory findings include plasma sodium = 126 mmol/L; plasma potassium = 6.8 mmol/L; and plasma glucose = 5.9 mmol/ L. What is the most likely diagnosis?

pyloric stenosis

gastroenteritis

hyperaldosteronism

panhyperpituitarism

congenital adrenal hyperplasia

A

A- congenital adrenal hyperplasia

140
Q

A 15-year-old basketball player complains of pain in his knees. Clinical examination reveals, in addition to tenderness, a swollen and prominent tibial tubercle. X-rays of the knee are unremarkable. What is the most likely diagnosis?

Osgood-Schlatter’s disease

Popliteal cyst

Slipped capital femoral epiphysis

Legg-Calve-Perthes disease

Gonococcal arthritis

A

A- Osgood-Schlatter’s disease

141
Q

Two infants are born at 36 weeks’ gestation. One infant weighs 2600g at birth and an unrelated second infant weighs 1600g. Which of the following conditions is the second baby more likely to have?

congenital malformations

low haematocrit

hyperglycaemia

surfactant deficiency

occipito-frontal (head) circumference that is small compared to body weight

A

A- congenital malformations

SGA - hypoglycaemic, won’t have surfactant deficiency (due to stress in utero, will have a relatively large head (brain sparing blood supply), high haematocrit…may well have congenital malformations

142
Q

A ten-year-old girl presents with recurrent abdominal pain. Of the following symptoms, which is most suggestive of an organic aetiology?

Periumbilical location

She has lost touch with her friends at school

She wakes from sleep with pain at night

The pain has persisted intermittently for more than three months

The pain is so severe that she cannot attend school

A

A- She wakes from sleep with pain at night

143
Q

A 14-year-old boy is brought to the clinic because of tenderness of the right breast. Examination reveals a small, firm nodule, which feels like hypertrophied breast tissue, beneath the areola. What is the most likely explanation for this finding?

Hermaphrodism

Pseudohermaphrodism

physiological gynaecomastia

an oestrogen-secreting adrenal tumour

an androgen-secreting adrenal tumour

A

A- physiological gynaecomastia

144
Q

A 6-year-old girl is brought to the GP with multiple, dark lesions on her legs and arms for the past month. She is afebrile and systemically well. The lesions are 1cm in diameter, nonpalpable, nontender, and not pruritic. She had spent the summer with her family in rural Thailand. What is the most likely diagnosis?

Erythema multiforme

post-inflammatory hyperpigmentation

Henoch-Schonlein purpura

Dengue infection

Tinea corporis

A

A- post-inflammatory hyperpigmentation

Hyperpigmented areas on arms and legs in child returning from Thailand. Child is well. 1cm, non-palpable, non-tender, non-pruritic…post inflammatory hyper pigmentation,

Dengue - fever, pruritic rash, painful (break-bone fever)

Hennoch-Schönlein - pruritic vasculitic rash, IgA mediated nephritis, legs arms and buttocks.

Erythema multiforme - multiform red blotches, with target lesions and mucus membrane involvement (this plus conjunctivitis, urethritis? Stevens-Johnson syndrome)

145
Q

A 16-month-old boy is not yet walking unaided. He has “bottom-shuffled” from the age of 10 months. His vision, hearing and fine motor skills are within normal limits. He has an older brother who had normal milestones. What is the best action to take next?

measure his creatine kinase (CK) level

refer him for physiotherapy

refer him for assessment by the Child Development Team

send blood for chromosomal analysis

review him in 3 months

A

A- review him in 3 months

Bottom-shufflers tend to walk later. 16-month old bottom shuffler not yet walking unaided - review in 3 months

146
Q

A 6 year-old Nigerian girl is seen in A&E with a history of fever and malaise a few days earlier, and an emerging rash. The rash first started on the face as an intense flushing but then spread to the trunk and extremities. On the upper arms it developed a lacy reticular appearance. The girl is afebrile and not ill looking. Which of the following complications is this girl at possible risk of developing?

Deafness

cerebellar ataxia

aplastic crisis

glomerulo-nephritis

secondary bacterial infection

A- aplastic crisis

A

A- aplastic crisis

147
Q

For the following clinical scenarios, choose the single most likely underlying cause from the list of options. Each option may be used once, more than once or not at all.

A 3-month-old breast-fed girl presents with a 3 day history of increasing difficulty with feeding. On examination she is tachypnoeic / hypoxic with no crepitations or wheeze and no abnormal upper airway signs.

Tuberculosis

Mumps

Chicken pox

Influenza

Bacterial meningitis

Meningococcal sepsis

HIV/AIDS

Measles

Malaria

Respiratory syncitial virus

A

A- HIV/AIDS

RSV and influenza are wet coryzal illnesses, which you can hear!

Hypoxia and tachypnoea with feeding difficulties, but nothing to hear on auscultation - pneumocystis pneumonia in AIDS/HIV is an interstitial pneumonia which would give these signs.

148
Q

For the following clinical scenarios, choose the single most likely underlying cause from the list of options. Each option may be used once, more than once or not at all.

A 6-year-old South African boy, domiciled in the UK, has just returned from visiting his family in Cape Town during July and August. He has a high fever with rigors, is coughing and is complaining of muscular aches.

Tuberculosis

Mumps

Chicken pox

Influenza

Bacterial meningitis

Meningococcal sepsis

HIV/AIDS

Measles

Malaria

Respiratory syncitial virus

A

A- Influenza

High fever and rigors with muscle pain in South African winter - influenza.

149
Q

For the following clinical scenarios, choose the single most likely underlying cause from the list of options. Each option may be used once, more than once or not at all.

A 14-year-old girl presents with a 5 day history of abdominal pain, submandibular and parotid gland enlargement, low grade fever and severe headaches. She has not received any antibiotics. On examination she is photophobic with neck stiffness, but is well perfused. She has no rash. On lumbar puncture the cerebrospinal fluid has normal protein and glucose, 50 WBC/ml (95% lymphocytes) and no RBC.

Tuberculosis

Mumps

Chicken pox

Influenza

Bacterial meningitis

Meningococcal sepsis

HIV/AIDS

Measles

Malaria

Respiratory syncitial virus

A

Mumps

150
Q

For the following clinical scenarios, choose the single most likely underlying cause from the list of options. Each option may be used once, more than once or not at all.

A 6-year-old caucasian boy has a three-week history of fevers, not responding to oral antibiotics in the first and second weeks, increasing drowsiness and now presents with confusion. On examination he has a mildly stiff neck and his Glasgow Coma Score is 12. A lumbar puncture revealed raised protein (2.1 g/l.) in the cerebrospinal fluid, decreased glucose, 710 WBC/ml, no RBC and no organisms seen on Gram stain.

Tuberculosis

Mumps

Chicken pox

Influenza

Bacterial meningitis

Meningococcal sepsis

HIV/AIDS

Measles

Malaria

Respiratory syncitial virus

A

A- Tuberculosis

6 year old Caucasian male with 3 weeks of fevers not responding to abx - arrives with GCS 12, stiff neck, 710 WCC/ml in CSF (should be none, allow up to 5), no organisms on gram stain, raised protein (normal is 0.4g/L), low glucose, no red cells….

The low glucose tells us that it’s not viral. The symptoms and the high WCC tells us that this is meningitis. Very high protein, lack of response to abx and no growth tells us that this is TB meningitis.

This case is TB meningitis! Don’t have to be Asian to get TB…

Partially treated bacterial meningitis gives a mixture of white cells. In TB, this will be a Lymphocytosis. In bacterial meningitis there will be neutrophilia

151
Q

For the following clinical scenarios, choose the single most likely underlying cause from the list of options. Each option may be used once, more than once or not at all.

A 5-year-old girl who never received MMR was exposed to chicken pox last week at school. She now presents with a 12 hour history of rash and abdominal pain and is reluctant to walk. On examination, she is unwell with cold hands and feet, and a widespread blanching maculopapular rash with spots of different sizes sparing the head and neck.

Tuberculosis

Mumps

Chicken pox

Influenza

Bacterial meningitis

Meningococcal sepsis

HIV/AIDS

Measles

Malaria

Respiratory syncitial virus

A

A- Meningococcal sepsis

Cold hands and feet and unwell - shock until proven otherwise! SEPSIS (bacterial? G-ive?

Note that meningococcal rashes have to start somehow! Start as blanching but then becomes non-blanching petechiae rash. Often develop some myosotis too, meaning that they are reluctant to walk. Possibly due to venous thrombosis in capillary beds and within muscles (muscle and abdominal pain). Note also the in the shock the blood is being diverted to the blood and kidneys.

Measles - starts with coryza and conjunctivitis. Rash spreads from face down to trunk and peripheries. Most commonly starts behind the ears. Won’t cause shock in the acute phase (maybe if get severe secondary bacterial infection)

(exampthon - a rash on the outside of the body. enampthon - inside of the body e.g. Koplik spots on mucus membranes! Very early phase only, last 24hrs. Get both of these in measles)

Petechial rash in only the head and neck - due to e.g. Forceful coughing, vomiting etc I.e. Increased venous pressure in the SVC region. E.g. In Bordatella pertussis infection. Includes subconjunctival haemorrhage.

152
Q

For the following clinical scenarios, choose the single most likely underlying condition from the list of options. Each option may be used once, more than once or not at all.

A 5-week-old boy presented with a 5 day history of vomiting after feeds. He has always tended to posset. In the last 24 hours he appears anxious when feeding. The Red Book confirms a weight loss of 200g. since he was weighed two weeks ago.

Acquired immune deficiency (AIDS)

Cystic fibrosis

Iron deficiency anaemia

Coeliac disease

Cerebral palsy

Gastro-oesophageal reflux

Giardia lamblia infection

Congenital hypothyroidism

Pyloric stenosis

Hyperthyroidism

A

Pyloric stenosis

Posseting - bringing up a mouthful of milk after a feed.

153
Q

For the following clinical scenarios, choose the single most likely underlying condition from the list of options. Each option may be used once, more than once or not at all.

A 10-month-old girl has grown along the 50th centile for weight until around 6 months but is now on the 25th centile. The mother has noticed a distended abdomen and says her stools are more frequent and bulky.

Acquired immune deficiency (AIDS)

Cystic fibrosis

Iron deficiency anaemia

Coeliac disease

Cerebral palsy

Gastro-oesophageal reflux

Giardia lamblia infection

Congenital hypothyroidism

Pyloric stenosis

Hyperthyroidism

A

A- Coeliac disease

154
Q

For the following clinical scenarios, choose the single most likely underlying condition from the list of options. Each option may be used once, more than once or not at all.

A 7-month-old girl adopted from an orphanage in Nepal is below 0.4th centile for length and weight. The birth history was apparently normal, apart from prolonged jaundice postnatally. She is not yet sitting. On examination she is hypotonic and flaccid. The lower limb reflexes are hard to elicit.

Acquired immune deficiency (AIDS)

Cystic fibrosis

Iron deficiency anaemia

Coeliac disease

Cerebral palsy

Gastro-oesophageal reflux

Giardia lamblia infection

Congenital hypothyroidism

Pyloric stenosis

Hyperthyroidism

A

A- Congenital hypothyroidism

155
Q

For the following clinical scenarios, choose the single most likely underlying condition from the list of options. Each option may be used once, more than once or not at all.

A 5-year-old-girl has recently arrived in the UK from Ukraine. She has a recent history of persistent loose stools associated with growth faltering. She has intermittent bloating and abdominal discomfort but on examination there are no other abnormal findings. Investigations reveal a negative Ttg test.

Acquired immune deficiency (AIDS)

Cystic fibrosis

Iron deficiency anaemia

Coeliac disease

Cerebral palsy

Gastro-oesophageal reflux

Giardia lamblia infection

Congenital hypothyroidism

Pyloric stenosis

Hyperthyroidism

A

A- Giardia lamblia infection

156
Q

For the following clinical scenarios, choose the single most likely underlying condition from the list of options. Each option may be used once, more than once or not at all.

A 2-year-old boy from a socially disadvantaged family presents with dental caries and frequent upper respiratory tract infections. He is a very fussy eater. On examination he has chronic suppurative otitis media but is avidly sucking from a bottle of cow’s milk. He is continuing to grow along the 25th centile for height and weight.

Acquired immune deficiency (AIDS)

Cystic fibrosis

Iron deficiency anaemia

Coeliac disease

Cerebral palsy

Gastro-oesophageal reflux

Giardia lamblia infection

Congenital hypothyroidism

Pyloric stenosis

Hyperthyroidism

A

A- Iron deficiency anaemia

Prop feeding I.e. milk bottle when lying down - risk of dental caries and chronic suppurative otitis media

157
Q

For the following clinical scenarios, choose the single most likely underlying cause from the list of options. Each option may be used once, more than once or not at all.

A 6-week-old breast-fed infant presents with a prolonged history of vomiting after feeds. He is growing along the 0.4th centile for weight. A pyloric mass is not palpable during a test feed. The plasma chloride is 102 mmol/l (NR 96-110).

Pyloric stenosis

Food allergy

Galactosaemia

Urinary tract infection

Gastro-oesophageal reflux

Malrotation

Congenital adrenal hyperplasia

Meningitis

Lactose intolerance

Encephalitis

A

A- Gastro-oesophageal reflux

158
Q

For the following clinical scenarios, choose the single most likely underlying cause from the list of options. Each option may be used once, more than once or not at all.

A one-week-old child presents with vomiting, lethargy, increasing jaundice. On examination a 3 cm liver is palpable. Ophthalmoscopy revelas that the red reflexes are absent.

Pyloric stenosis

Food allergy

Galactosaemia

Urinary tract infection

Gastro-oesophageal reflux

Malrotation

Congenital adrenal hyperplasia

Meningitis

Lactose intolerance

Encephalitis

A

A- Galactosaemia

159
Q

For the following clinical scenarios, choose the single most likely underlying cause from the list of options. Each option may be used once, more than once or not at all.

A 3-year-old child presents with acute onset abdominal pain for 4 hours. She has had 3 similar episodes in the past, which were all self-resolving. This time the vomiting is bile-stained.

Pyloric stenosis

Food allergy

Galactosaemia

Urinary tract infection

Gastro-oesophageal reflux

Malrotation

Congenital adrenal hyperplasia

Meningitis

Lactose intolerance

Encephalitis

A

A- Malrotation

160
Q

For the following clinical scenarios, choose the single most likely underlying cause from the list of options. Each option may be used once, more than once or not at all.

A 6-month-old breast-fed infant has developed vomiting since early life, which has not resolved with anti-reflux medication. He also has eczema, which is hard to control.

Pyloric stenosis

Food allergy

Galactosaemia

Urinary tract infection

Gastro-oesophageal reflux

Malrotation

Congenital adrenal hyperplasia

Meningitis

Lactose intolerance

Encephalitis

A

A- Food allergy

161
Q

For the following clinical scenarios, choose the single most likely underlying cause from the list of options. Each option may be used once, more than once or not at all.

A 15-month-old girl presents with a 3-day history of intermittent fevers and vomiting and poor feeding. On examination her temperature is 38.6 C but there are no localising signs.

Pyloric stenosis

Food allergy

Galactosaemia

Urinary tract infection

Gastro-oesophageal reflux

Malrotation

Congenital adrenal hyperplasia

Meningitis

Lactose intolerance

Encephalitis

A

A- Urinary tract infection

162
Q

For the following clinical scenarios, choose the single most likely underlying cause from the list of options. Each option may be used once, more than once or not at all.

5 yr old with 1 week of severe left sided headaches, analgesic unresponsive. Having high fevers, vomiting, and earache, on oral amoxicillin for otitis media. Aside from a red left TM, nothing to find on examination

Mastoiditis

Idiopathic intracranial hypertension

Posterior fossa tumour

Hypertension

Right hemispheric tumour

Arterio-venous malformation

Tension type headaches

Intracranial abscess

Chronic sinusitis

Migraine

A

A- Intracranial abscess

163
Q

For the following clinical scenarios, choose the single most likely underlying cause from the list of options. Each option may be used once, more than once or not at all.

A 7 year old child presents to A+E with a severe sudden onset headache and neck stiffness, and is noted to have a positive Kernig’s sign. He has had several episodes of acute onset severe right sided headaches in the past with left sided weakness, which fully resolved within an hour. These episodes have recently become more severe and the weakness has taken longer to resolve.

Mastoiditis

Idiopathic intracranial hypertension

Posterior fossa tumour

Hypertension

Right hemispheric tumour

Arterio-venous malformation

Tension type headaches

Intracranial abscess

Chronic sinusitis

Migraine

A

A- Arterio-venous malformation

A-V malformations may present with what look like TIAs and strokes. Will have sudden onset severe headache, weakness etc I,e. Looks like a stroke. If haemorrhagic, will have meningism e.g. Kernig’s sign. LP will uniformly have lots of RBCs, different to just a bloody tap.

164
Q

For the following clinical scenarios, choose the single most likely underlying cause from the list of options. Each option may be used once, more than once or not at all.

3 yr old with 2 week history of waking with headaches and vomiting; becoming unsteady on feet and presents with a sudden squint of left eye, with diplopia on looking to the left.

Mastoiditis

Idiopathic intracranial hypertension

Posterior fossa tumour

Hypertension

Right hemispheric tumour

Arterio-venous malformation

Tension type headaches

Intracranial abscess

Chronic sinusitis

Migraine

A

A- Posterior fossa tumour

165
Q

For the following clinical scenarios, choose the single most likely underlying cause from the list of options. Each option may be used once, more than once or not at all.

9 yr old with 6-12 months of continuous daily frontal and vertex headaches; unresponsive to analgesia; does not wake from sleep; no interruption to activities of daily living. No other symptoms. Examination is normal, including blood pressure and fundi.

Mastoiditis

Idiopathic intracranial hypertension

Posterior fossa tumour

Hypertension

Right hemispheric tumour

Arterio-venous malformation

Tension type headaches

Intracranial abscess

Chronic sinusitis

Migraine

A

A- Tension type headaches

166
Q

For the following clinical scenarios, choose the single most likely underlying cause from the list of options. Each option may be used once, more than once or not at all.

14 year old obese girl presents with a 6 week history of worsening headaches, which occur on waking and are associated vomiting. She has grossly blurred fundi. Her CT scan normal.

Mastoiditis

Idiopathic intracranial hypertension

Posterior fossa tumour

Hypertension

Right hemispheric tumour

Arterio-venous malformation

Tension type headaches

Intracranial abscess

Chronic sinusitis

Migraine

A

A- Idiopathic intracranial hypertension

167
Q

For the following clinical scenarios, choose the single investigation most likely to reveal the underlying diagnosis. Each option may be used once, more than once or not at all.

A 5 year old who has recently returned from Nigeria, has had 3 days of swinging high fevers. His urine is noted to be dark red. On examination he has enlarged liver, but no other abnormal findings. A dipstick is positive for blood +++.

Von Willebrand’s factor assay

Blood cultures

G6PD level, now and in one month’s time

Urine M, C & S

24 hour urine protein quantification

Urinary schistosomiasis antigen

Ultrasound scan of abdomen

Complement C3 and C4 levels

PT and APTT

Thin and thick film

A

A- Thin and thick film

168
Q

For the following clinical scenarios, choose the single investigation most likely to reveal the underlying diagnosis. Each option may be used once, more than once or not at all.

A 2 year old presents with frank haematuria, some abdominal pain and rigors.

Von Willebrand’s factor assay

Blood cultures

G6PD level, now and in one month’s time

Urine M, C & S

24 hour urine protein quantification

Urinary schistosomiasis antigen

Ultrasound scan of abdomen

Complement C3 and C4 levels

PT and APTT

Thin and thick film

A

A- Urine M, C & S

169
Q

For the following clinical scenarios, choose the single investigation most likely to reveal the underlying diagnosis. Each option may be used once, more than once or not at all.

A 6 month old is examined by the GP because of parental concern about constipation over the last 6 weeks. On examination he is well and thriving and has a palpable right sided abdominal mass. He has microscopic haematuria.

Von Willebrand’s factor assay

Blood cultures

G6PD level, now and in one month’s time

Urine M, C & S

24 hour urine protein quantification

Urinary schistosomiasis antigen

Ultrasound scan of abdomen

Complement C3 and C4 levels

PT and APTT

Thin and thick film

A

A- ultrasound scan of the abdomen

170
Q

For the following clinical scenarios, choose the single investigation most likely to reveal the underlying diagnosis. Each option may be used once, more than once or not at all.

A 3 year old girl’s parents have noted that her urine has gone very dark and she has had puffy eyes. She has been unwell recently with a fever and sore throat. Urine shows blood ++++, protein +++. BP 110/65.

Von Willebrand’s factor assay

Blood cultures

G6PD level, now and in one month’s time

Urine M, C & S

24 hour urine protein quantification

Urinary schistosomiasis antigen

Ultrasound scan of abdomen

Complement C3 and C4 levels

PT and APTT

Thin and thick film

A

Complement C3 and C4 levels

Nephritic syndrome - HTN, haematuria, proteinuria, oedema. Nephritis is often post-streptococcal - immune mediated process, complement deposits in the kidneys, so would find c3 and c4 to be low. Most docs wouldn’t do a biopsy here!

(Nephrotic syndrome - e.g. Minimal change disease)

171
Q

For the following clinical scenarios, choose the single investigation most likely to reveal the underlying diagnosis. Each option may be used once, more than once or not at all

A 6 yr old male, UK born, of Iraqi parents, develops sudden onset very dark red urine, no pain. He has a current viral infection. On exam, slightly jaundiced, no liver or spleen; urine – blood ++++. There was a history of prolonged neonatal jaundice.

Von Willebrand’s factor assay

Blood cultures

G6PD level, now and in one month’s time

Urine M, C & S

24 hour urine protein quantification

Urinary schistosomiasis antigen

Ultrasound scan of abdomen

Complement C3 and C4 levels

PT and APTT

Thin and thick film

A

A- G6PD level, now and in one month’s time

172
Q

For the following clinical scenarios, choose the single most appropriate treatment from the list of options. Each option may be used once, more than once or not at all.

A 2-year-old-boy presents to A&E with a generalised itchy rash, stridor and tingling of his mouth one hour after eating a Snickers bar.

Atropine

Prednisolone orally

Adrenaline (epinephrine)

No treatment - admit for observation

i.v. Salbutamol

Paraldehyde

Aminophylline

Desferrioxamine

Procyclidine

N-acetylcysteine

A

A- Adrenaline (epinephrine)

173
Q

For the following clinical scenarios, choose the single most appropriate treatment from the list of options. Each option may be used once, more than once or not at all.

A 15 year-old girl was brought to the A&E after disclosing to her teacher at school that she had taken about 40 tablets the evening before. On arrival she was alert but complaining of nausea and right upper quadrant abdominal pain. Bloods taken on arrival at A&E showed a raised ALT and AST and abnormal coagulation studies.

Atropine

Prednisolone orally

Adrenaline (epinephrine)

No treatment - admit for observation

i.v. Salbutamol

Paraldehyde

Aminophylline

Desferrioxamine

Procyclidine

N-acetylcysteine

A

a- N-acetylcysteine

174
Q

For the following clinical scenarios, choose the single most appropriate treatment from the list of options. Each option may be used once, more than once or not at all.

A 7 year old girl with severe wheeze, a peak expiratory flow rate of 40% and a heart rate of 120 bpm presented to A+E an hour ago. She has been treated with burst therapy using Salbutamol and Ipratroprium Bromide but has had no response.

Atropine

Prednisolone orally

Adrenaline (epinephrine)

No treatment - admit for observation

i.v. Salbutamol

Paraldehyde

Aminophylline

Desferrioxamine

Procyclidine

N-acetylcysteine

A

A- i.v. Salbutamol

O SHIT MAN - acute severe asthma attack

oxygen

Salbutamol

Hydrocortisone

Ipratropium bromide

Theophylline/amyophilline

Magnesium sulphate

Anaesthetist

175
Q

For the following clinical scenarios, choose the single most appropriate treatment from the list of options. Each option may be used once, more than once or not at all.

A 3-year-old boy with diarrhoea and vomiting was given metoclopromide and loperamide. He presents to A&E with torticollis and an intermittent convergent squint and upward deviation of both eyes.

Atropine

Prednisolone orally

Adrenaline (epinephrine)

No treatment - admit for observation

i.v. Salbutamol

Paraldehyde

Aminophylline

Desferrioxamine

Procyclidine

N-acetylcysteine

A

A- Procyclidine

Oculogyric crisis - dystonic reaction. intermittent convergent squint and upward deviation of both eyes. Not a seizure, child is fully conscious throughout. In this case, it was caused by giving metoclopromide and loperamide for D&V.

Procyclidine be used to treat this.

Torticollis is twisting of the neck to one side.

176
Q

For the following clinical scenarios, choose the single most appropriate treatment from the list of options. Each option may be used once, more than once or not at all.

a 3 year old girl has been brought into A+E after drinking about 100 ml of Ibuprofen (100 mg/5 ml) suspension.

Atropine

Prednisolone orally

Adrenaline (epinephrine)

No treatment - admit for observation

i.v. Salbutamol

Paraldehyde

Aminophylline

Desferrioxamine

Procyclidine

N-acetylcysteine

A

A- No treatment - admit for observation

177
Q

For each of the following genetic disorders, select the most likely mode of inheritance from the options. Each option may be used once, more than once, or not at all

A 3-year-old boy is below the 0.4th centile for his height. He has abnormal arms and legs and his head appears large. He has a lumbar lordosis.

X-linked dominant

Trisomy

Trinucleotide expansions

X-linked recessive

Point deletion

Autosomal dominant

Polygenic

Uniparental disomy

Autosomal recessive

Mitochondrial inheritance

A

A- Autosomal dominant

178
Q

For each of the following genetic disorders, select the most likely mode of inheritance from the options. Each option may be used once, more than once, or not at all

A 2-year-old girl was referred by her GP to the outpatient department. She has been complaining of intermittent abdominal pain and generalised aches and pains for the last 3 days. She is pale, febrile and icteric. She has a soft flow murmur and splenomegaly. There is swelling of her wrists and tender swelling of her fingers.

X-linked dominant

Trisomy

Trinucleotide expansions

X-linked recessive

Point deletion

Autosomal dominant

Polygenic

Uniparental disomy

Autosomal recessive

Mitochondrial inheritance

A
  • Autosomal recessive

Sickle cell disease - swaps valine for glutamine - autosomal recessive. Note that any condition that has ‘carriers’ is autosomal recessive.

179
Q

For each of the following genetic disorders, select the most likely mode of inheritance from the options. Each option may be used once, more than once, or not at all

A 7-year-old boy presents to A&E with a painful right elbow with limited range of movement and a bluish discolouration to the overlying skin. Laboratory findings include WCC=9.6 x 109/L with normal differential; platelet = 210 x 109/L; ESR=12mm/hr; CRP=6g/ml. PT and bleeding time is normal. APTT is elevated.

X-linked dominant

Trisomy

Trinucleotide expansions

X-linked recessive

Point deletion

Autosomal dominant

Polygenic

Uniparental disomy

Autosomal recessive

Mitochondrial inheritance

A

A- X-linked recessive

180
Q

For each of the following genetic disorders, select the most likely mode of inheritance from the options. Each option may be used once, more than once, or not at all

A 6-day-old baby girl was born at term via an elective Caesarian section for breech presentation. A check ultrasound scan revealed a dislocated left hip as delivery was difficult. Her mother and older sister had similar problems in infancy.

X-linked dominant

Trisomy

Trinucleotide expansions

X-linked recessive

Point deletion

Autosomal dominant

Polygenic

Uniparental disomy

Autosomal recessive

Mitochondrial inheritance

A

A- Polygenic

181
Q

For each of the following genetic disorders, select the most likely mode of inheritance from the options. Each option may be used once, more than once, or not at all

An 8-month-old boy was referred to the outpatient clinic by the Health Visitor who noted that he was floppy and failing to gain weight. Examination revealed a hypotonic infant with a flattened face, slanted palpebral fissures, speckled irises and a grade 3/6 pansystolic murmur best heard at the lower left sternal border

X-linked dominant

Trisomy

Trinucleotide expansions

X-linked recessive

Point deletion

Autosomal dominant

Polygenic

Uniparental disomy

Autosomal recessive

Mitochondrial inheritance

A

Trisomy

182
Q

For each clinical scenario, select the most likely diagnosis from the list of options. Each option may be used once, more than once, or not at all.

A term infant delivered by elective Caesarian section develops tachypnoea, grunting and recession at 30 minutes of age. A CXR shows flattened diaphragms with fluid in the right horizontal fissure and well-aerated lung fields. The infant improves after 4 hours and no longer requires oxygen.

Primary pulmonary hypertension

Asthma

Respiratory distress syndrome

Meconium aspiration

Pulmonary interstitial emphysema

Pneumothorax

Bronchiolitis

Bacterial pneumonia

Transient tachypnoea of the newborn

Chronic lung disease

A

A- Transient tachypnoea of the newborn

183
Q

For each clinical scenario, select the most likely diagnosis from the list of options. Each option may be used once, more than once, or not at all.

A post-term infant is delivered by emergency Caesarian section for a moderate placental abruption. On examination the infant’s oxygen saturation is 85% in air and has marked respiratory distress. A CXR reveals bilateral patchy infiltrates.

Primary pulmonary hypertension

Asthma

Respiratory distress syndrome

Meconium aspiration

Pulmonary interstitial emphysema

Pneumothorax

Bronchiolitis

Bacterial pneumonia

Transient tachypnoea of the newborn

Chronic lung disease

A

A- Meconium aspiration

Post term child delivered by c-section for partial placental disruption with respiratory distress, low oxygen saturations and patchy bilateral infiltrates on CXR - bowels open in utero in stressful conditions - this is meconium aspiration!

184
Q

For each clinical scenario, select the most likely diagnosis from the list of options. Each option may be used once, more than once, or not at all.

An infant of 31 weeks gestation was born via an emergency section because of foetal decelerations noted on CTG. The birth weight was 1100g and the infant required intubation but was difficult to ventilate. A CXR shows diffuse whiteout of both lungs with an air bronchogram.

Primary pulmonary hypertension

Asthma

Respiratory distress syndrome

Meconium aspiration

Pulmonary interstitial emphysema

Pneumothorax

Bronchiolitis

Bacterial pneumonia

Transient tachypnoea of the newborn

Chronic lung disease

A

A- Respiratory distress syndrome

Respiratory distress syndrome shows diffuse whiteout on CXR with an air bronchogram. Occurs in preterms.

185
Q

For each clinical scenario, select the most likely diagnosis from the list of options. Each option may be used once, more than once, or not at all.

A preterm infant is now 7 weeks old. She has been extubated for 10 days but is still oxygen dependent. Her CXR now shows patchy infiltrates with areas of lucency. She is currently on diuretics.

Primary pulmonary hypertension

Asthma

Respiratory distress syndrome

Meconium aspiration

Pulmonary interstitial emphysema

Pneumothorax

Bronchiolitis

Bacterial pneumonia

Transient tachypnoea of the newborn

Chronic lung disease

A

A- Chronic lung disease

CLD cxr shows patchy infiltrates with areas of lucency.

186
Q

For each clinical scenario, select the most likely diagnosis from the list of options. Each option may be used once, more than once, or not at all.

A term baby is born normally at 38 weeks gestation at 2.3 kg. Labour was difficult with prolonged rupture of membranes. At 35 minutes of age the infant was noted to have an increasing oxygen requirement with grunting and respiratory distress. On examination he was floppy with an oxygen saturation of 95% in 2L/min of oxygen. A CXR showed reticulonodular shadowing.

Primary pulmonary hypertension

Asthma

Respiratory distress syndrome

Meconium aspiration

Pulmonary interstitial emphysema

Pneumothorax

Bronchiolitis

Bacterial pneumonia

Transient tachypnoea of the newborn

Chronic lung disease

A

A- Bacterial pneumonia

Bacterial pneumonia can occur if there are risk factors of sepsis in neonates.

187
Q

For each of the following clinical scenarios, select the most likely immunological mediator from the list of options. Each option may be used once, more than once or not at all.

A 4-month-old boy who is well and thriving has blood test results positive for HIV antibodies.

IgG antibodies

T cells

IgA antibodies

Eosinophils

Complement C3

Complement C4

C1 esterase inhibitor

IgE antibodies

Neutrophils

IgM antibodies

A

A- IgG antibodies

Baby has positive HIV antibody test - these will be maternal antibodies - therefore this is IgG mediated, as IgA and IgM are too big!

(If IgM is found in the baby, it has come from the baby!)

If e baby is uninflected by HIV, can detect these antibodies for up to 18 months. In the case of hepatitis C, will have maternal antibodies up to 14 months. Note that this is a financial cause, because the HIV assay is so much more sensitive!

(HIV can be transmitted by breast feeding)

188
Q

For each of the following clinical scenarios, select the most likely immunological mediator from the list of options. Each option may be used once, more than once or not at all.

A 14-year-old girl is stung by a bee and develops urticaria within 20 minutes of the sting.

IgG antibodies

T cells

IgA antibodies

Eosinophils

Complement C3

Complement C4

C1 esterase inhibitor

IgE antibodies

Neutrophils

IgM antibodies

A
189
Q

For each of the following clinical scenarios, select the most likely immunological mediator from the list of options. Each option may be used once, more than once or not at all.

An 18-month-old boy is seen with a history of recurrent oral candidiasis. He developed tetany in the newborn period and had cardiac surgery for a complex heart defect.

IgG antibodies

T cells

IgA antibodies

Eosinophils

Complement C3

Complement C4

C1 esterase inhibitor

IgE antibodies

Neutrophils

IgM antibodies

A

A- T cells

DiGeorge syndrome - lack of thymus! Had tetany in newborn period and cardiac surgery….(clues for digeorge’s). Presented with oral candidiasis recurrence as haven’t got proper T-cells.

190
Q

For each of the following clinical scenarios, select the most likely immunological mediator from the list of options. Each option may be used once, more than once or not at all.

An 8-year-old girl is seen with second episode of cervical abscess in 3 months. The first culture grew Klebsiella pneumoniae. On this occasion Staphylococcus aureus was cultured.

IgG antibodies

T cells

IgA antibodies

Eosinophils

Complement C3

Complement C4

C1 esterase inhibitor

IgE antibodies

Neutrophils

IgM antibodies

A

A- Neutrophils

191
Q

For each of the following clinical scenarios, select the most likely immunological mediator from the list of options. Each option may be used once, more than once or not at all.

A 12-year-old boy has had seven episodes of spontaneous lip swelling and bilateral periorbital oedema in the last 3 years. His father also had similar episodes in childhood.

IgG antibodies

T cells

IgA antibodies

Eosinophils

Complement C3

Complement C4

C1 esterase inhibitor

IgE antibodies

Neutrophils

IgM antibodies

A
192
Q

For each of the following patients please select the most likely diagnosis from the list of options. Each option may be used once, more than once, or not at all.

A four year old boy presents with chronic cough, poor weight gain, loose stools and finger clubbing.

Gastro oesophagus reflux

Pulmonary TB

Pertussis

Chronic asthma

Obstructive sleep apnea

Epiglotittis

Lobar Pneumonia

Viral Croup

Cystic Fibrosis

Tracheo-oesophageal fistula

A
193
Q

For each of the following patients please select the most likely diagnosis from the list of options. Each option may be used once, more than once, or not at all.

A five week old girl presents with a history of recurrent coughing and choking during and after feeds. Pregnancy was complicated by polyhydramnios.

Gastro oesophagus reflux

Pulmonary TB

Pertussis

Chronic asthma

Obstructive sleep apnea

Epiglotittis

Lobar Pneumonia

Viral Croup

Cystic Fibrosis

Tracheo-oesophageal fistula

A
194
Q

For each of the following patients please select the most likely diagnosis from the list of options. Each option may be used once, more than once, or not at all.

An eight year old boy presents with nocturnal cough and early morning tiredness. He has a past history of serous otitis media. On examination, he has noisy breathing, is overweight and is inattentive during consultation.

Gastro oesophagus reflux

Pulmonary TB

Pertussis

Chronic asthma

Obstructive sleep apnea

Epiglotittis

Lobar Pneumonia

Viral Croup

Cystic Fibrosis

Tracheo-oesophageal fistula

A
195
Q

For each of the following patients please select the most likely diagnosis from the list of options. Each option may be used once, more than once, or not at all.

A five month old baby boy presents with a three week history of persistent cough with no wheeze. Sometimes he vomits after coughing. He has had two doses of the primary immunisations.

Gastro oesophagus reflux

Pulmonary TB

Pertussis

Chronic asthma

Obstructive sleep apnea

Epiglotittis

Lobar Pneumonia

Viral Croup

Cystic Fibrosis

Tracheo-oesophageal fistula

A

A- Pertussis

196
Q

For each of the following patients please select the most likely diagnosis from the list of options. Each option may be used once, more than once, or not at all.

A three year old girl presents with a two day history of fever of 37.8º and stridor. She has vomited twice prior to being seen in Accident & Emergency.

Gastro oesophagus reflux

Pulmonary TB

Pertussis

Chronic asthma

Obstructive sleep apnea

Epiglotittis

Lobar Pneumonia

Viral Croup

Cystic Fibrosis

Tracheo-oesophageal fistula

A
197
Q

For the following clinical scenarios choose the most appropirate action from the list of options. Each option may be used once, more than once or not at all.

A two year old boy is brought into A+E by ambulance having a generalised tonic-clonic seizure. Over the last 24 hours he has had a slightly runny nose, and his mum has been giving him regular Paracetamol for a low grade fever. Mum also comments that she herself had a cold sore recently. On arrival at A+E he is still having the seizure.

organise neuro-imaging

reassure the parents

do a blood gas

start aciclovir

give rectal diazepam

do a lumbar puncture

give sodium valproate

give intravenous phenytoin

start ceftriaxone

organise an EEG

A

A- give rectal diazepam

198
Q

For the following clinical scenarios choose the most appropriate action from the list of options. Each option may be used once, more than once or not at all.

A three month old baby has been admitted to the ward with a history of fever and a two minute generalised, self-terminating tonic-clonic seizure. She has had a lumbar puncture that shows white blood count 850 per mm3 (80% polymorphs), protein 1.2 g/l, glucose 1.7 mmol/l (blood glucose 5.1 mmol/l).

organise neuro-imaging

reassure the parents

do a blood gas

start aciclovir

give rectal diazepam

do a lumbar puncture

give sodium valproate

give intravenous phenytoin

start ceftriaxone

organise an EEG

A
199
Q

For the following clinical scenarios choose the most appropirate action from the list of options. Each option may be used once, more than once or not at all.

A 3 year old girl is brought into A+E by her parents. She is usually fit and well. Her mum says that half an hour ago she ran into a table, and banged her head on the corner. She went pale and fell to the floor, and then had two or three twitching movements before starting to cry. She is now back to her normal self and is running round A+E. Clinical examination is normal.

organise neuro-imaging

reassure the parents

do a blood gas

start aciclovir

give rectal diazepam

do a lumbar puncture

give sodium valproate

give intravenous phenytoin

start ceftriaxone

organise an EEG

A
200
Q

For the following clinical scenarios choose the most appropirate action from the list of options. Each option may be used once, more than once or not at all.

A seven year old girl who has previously been fit and well presents in A+E. Over the last few weeks her school teacher has commented that she has not been doing as well as normal. She has also been complaining of headaches. Today at school she had a left-sided seizure that lasted approximately three minutes.

organise neuro-imaging

reassure the parents

do a blood gas

start aciclovir

give rectal diazepam

do a lumbar puncture

give sodium valproate

give intravenous phenytoin

start ceftriaxone

organise an EEG

A

A- organise neuro-imaging

201
Q

For the following clinical scenarios choose the most appropirate action from the list of options. Each option may be used once, more than once or not at all.

A four year old known epileptic is admitted to the ward for observation as he has been vomiting. He is on regular sodium valproate. You are called urgently to the ward to see him. An hour ago he had his regular sodium valproate but vomited soon afterwards. He is now having a seizure and has been fitting for about six minutes. A fingerprick blood glucose is 4.7 mmol/l.

organise neuro-imaging

reassure the parents

do a blood gas

start aciclovir

give rectal diazepam

do a lumbar puncture

give sodium valproate

give intravenous phenytoin

start ceftriaxone

organise an EEG

A
202
Q

For the following clinical scenarios select the most appropirate diagnosis from the list of options. Each option may be used once, more than once or not at all.

A term neonate is cyanotic and tachypnoeic at birth. There is a soft pan systolic murmur at the lower sternal edge. An antenatal scan had shown a VSD, overriding aorta and infandibular stenosis.

interruption of the aortic arch

VSD

infective endocarditis

rheumatic fever

PDA

transposition of the great arteries

aortic coarctation

ASD

innocent murmur

tetralogy of Fallot

A

tetralogy of Fallot

203
Q

For the following clinical scenarios select the most appropirate diagnosis from the list of options. Each option may be used once, more than once or not at all.

A one week old term baby presents to A+E. He is tachypnoeic, tachycardic and has an enlarged liver. Feeding and weight gain have been poor. On examination the infact is acyanotic with a quiet systolic murmur best heard at the upper left sternal edge. The second heart sound is widely split, the split not varying with respiration.

interruption of the aortic arch

VSD

infective endocarditis

rheumatic fever

PDA

transposition of the great arteries

aortic coarctation

ASD

innocent murmur

tetralogy of Fallot

A
204
Q

For the following clinical scenarios select the most appropirate diagnosis from the list of options. Each option may be used once, more than once or not at all.

A ten year old girl is seen in the endocrine clinic for short stature. On examination she is noted to have widely spaced nipples, a webbed neck, cubitus valgus and an ejection systolic murmur at the upper sternal edge.

interruption of the aortic arch

VSD

infective endocarditis

rheumatic fever

PDA

transposition of the great arteries

aortic coarctation

ASD

innocent murmur

tetralogy of Fallot

A

Aortic coarctation

205
Q

For the following clinical scenarios select the most appropirate diagnosis from the list of options. Each option may be used once, more than once or not at all.

A two year old ex-preterm child, who recently arrived from Bhutan, has been admitted with pneumonia. He has had three episodes of lower respiratory tract infection in the past year. On examination a continuous murmur is heard beneath his left clavicle.

interruption of the aortic arch

VSD

infective endocarditis

rheumatic fever

PDA

transposition of the great arteries

aortic coarctation

ASD

innocent murmur

tetralogy of Fallot

A
206
Q

For the following clinical scenarios select the most appropirate diagnosis from the list of options. Each option may be used once, more than once or not at all.

A six year old child with a known VSD has had episodes of fever following a dental procedure. On examination she is febrile with a loud pan systolic murmur and a thrill. Her spleen tip is palpable. Her urine dipstick reveals haematuria.

interruption of the aortic arch

VSD

infective endocarditis

rheumatic fever

PDA

transposition of the great arteries

aortic coarctation

ASD

innocent murmur

tetralogy of Fallot

A

A- Infective endocarditis

207
Q

Molly is 12 months old and needs to be admitted to the paediatric ward in the district hospital. There is one paediatric ward in the hospital. Her mother is concerned whether they will be geared to caring for such a young child.

What is the most common age for children to be admitted to hospital?

Less than 1 year
1 – 3 years
3 – 5 years
5 – 10 years
10 – 16 years

A

Less than one year

The most common age is infants < 1 year old. Most medical admissions are emergencies in children under 5 years of age, whereas surgical admissions peak at 5 years of age, one-third of which are elective

208
Q

Rhys is a 5-year-old boy from Wales who is referred to the paediatric team as he has developed weakness in his legs. Investigations reveal he has Duchenne muscular dystrophy.

Who is the most appropriate person to inform the parents?

Sister on ward
Senior doctor
Junior doctor
Nurse looking after patient
General practitioner

A

Senior doctor

Parents say that they would prefer a senior doctor, in the presence of a nurse, breaking bad news to them.

209
Q

Daniel is a 15-year old boy from London who went to see his general practitioner as he has been tired and not ‘quite right’ for the last 2 months. The general practitioner took a full blood count to see if he was anaemic. The haematology laboratory phone you, a newly qualified doctor, at 6pm in the hospital saying they have received a full blood count on Daniel and his white cells are 200×109/L and that there are blast cells. You ring Daniel’s parents at home and tell them they need to come to the oncology ward at the hospital as the results of his blood tests are abnormal. They ask you what the abnormalities are and you tell them you will explain more when they come in. You ring the consultant who says he will come to the hospital to speak to the parents. When the parents arrive you ask them to wait 20 minutes until the consultant arrives and he will explain more.

What is not ideal about the situation?

You have told the parents over the phone the blood test was abnormal rather than in person

You have asked Daniel to come out of hours rather than the following morning

You did not answer the parent’s questions on the phone. You should have told them the diagnosis

The family had to wait for the consultant, you should have told them the diagnosis

You asked the family to come to an oncology ward rather than the paediatric assessment unit

A

You asked the family to come to an oncology ward rather than the paediatric assessment unit.

By telling them to come to an oncology ward you are telling them Daniel has cancer without actually explaining his diagnosis properly.

210
Q

Xu-Li is a 1-year-old with significant eczema. She has no other medical problems and is not on any medication.

Select the most appropriate way to administer the medication required.

Liquid
Intravenous
Intramuscular
Intradermal
Tablets
Inhaled
Nebulised
Topical
Subcutaneous

A

Topical

Emollients are the first line of therapy. These should be administered topically and should be applied regularly. The skin needs to be kept moist to prevent scratching.
A good way of assessing how often it is applied is to ask how quickly the pot of emollient was finished.

211
Q

For each of the scenarios below select the most appropriate way to administer the medication required. Each option may be used once, more than once, or not at all.

A mother has just given birth to a baby girl in a hospital in England. The midwife wants to give her vitamin K.

Liquid

IV

IM

Intradermal

Tablets

Inhaled

Nebulised

Topical

Subcutaenous

A

Intramuscular

212
Q

For each of the scenarios below select the most appropriate way to administer the medication required. Each option may be used once, more than once, or not at all.

Luke is a 3-year-old boy who is seen in the paediatric assessment unit. He is eating and drinking adequately. He has no other past medical history and is not on any medications. He has a fever which is associated with tachypnoea and crepitations at the right base. His oxygen saturation is 96% in air.

Liquid

IV

IM

Intradermal

Tablets

Inhaled

Nebulised

Topical

Subcutaenous

A

Liquid

213
Q

For each of the scenarios below select the most appropriate way to administer the medication required. Each option may be used once, more than once, or not at all.

Noel is a 7-day-old baby who is admitted to hospital with a fever. He is feeding 3–4 hourly from the breast and has no obvious source for his fever. He was born by vaginal delivery following a normal pregnancy.

Liquid

IV

IM

Intradermal

Tablets

Inhaled

Nebulised

Topical

Subcutaenous

A

IV

214
Q

For each of the scenarios below select the most appropriate way to administer the medication required. Each option may be used once, more than once, or not at all.

Mark is a 15-year-old boy who is seen in the emergency department with an exacerbation of wheeze. He is able to speak in short sentences but has an oxygen requirement and needs to be given salbutamol.

Liquid

IV

IM

Intradermal

Tablets

Inhaled

Nebulised

Topical

Subcutaenous

A

Nebuliser

215
Q

For each of the children in pain listed below, select the most appropriate next step in their pain management. Each option may be used once, more than once or not at all.

Victoria is 6 months old. She attends the outpatient department for a blood test.

Distraction

Topical Anaesthetic

Regular oral NSAIDs

IV Morphine via a nurse-controlled pump

IV patient controlled analgesia (PCA) with morphine

Regular oral paracetomol

Regular oral long-acting morphine with rapid action oromorph for breakthrough pain

Epidural

Intranasal diamorphine

Inhalation of nitrous oxide

Regular oral weak opioid e.g. codeine

A

Topical anaesthetic

216
Q

For each of the children in pain listed below, select the most appropriate next step in their pain management. Each option may be used once, more than once or not at all.

Zac is a 10-year-old boy who was diagnosed with a Ewing sarcoma. He has severe pain from metastatic disease, which is unresponsive to therapy. You ask the palliative care team to help with his management as the medications are insufficient. You have tried paracetamol and oral morphine without success. He has renal failure secondary to his chemotherapy. He has no allergies.

Distraction

Topical Anaesthetic

Regular oral NSAIDs

IV Morphine via a nurse-controlled pump

IV patient controlled analgesia (PCA) with morphine

Regular oral paracetomol

Regular oral long-acting morphine with rapid action oromorph for breakthrough pain

Epidural

Intranasal diamorphine

Inhalation of nitrous oxide

Regular oral weak opioid e.g. codeine

A

Regular oral long-acting morphine with rapid action oromorph for breakthrough pain

217
Q

For each of the children in pain listed below, select the most appropriate next step in their pain management. Each option may be used once, more than once or not at all.

Fiona is 3 years old. She has come from her home in Northern Ireland to England to have a liver transplant. She is day one post-operative.

Distraction

Topical Anaesthetic

Regular oral NSAIDs

IV Morphine via a nurse-controlled pump

IV patient controlled analgesia (PCA) with morphine

Regular oral paracetomol

Regular oral long-acting morphine with rapid action oromorph for breakthrough pain

Epidural

Intranasal diamorphine

Inhalation of nitrous oxide

Regular oral weak opioid e.g. codeine

A

IV Morphine via a nurse-controlled pump

218
Q

For each of the children in pain listed below, select the most appropriate next step in their pain management. Each option may be used once, more than once or not at all.

Achille is a 13-year-old black African boy who has sickle cell disease. He presents with pain in his left leg. He has already taken paracetamol without affect. He has no other medical problems and has no allergies.

Distraction

Topical Anaesthetic

Regular oral NSAIDs

IV Morphine via a nurse-controlled pump

IV patient controlled analgesia (PCA) with morphine

Regular oral paracetomol

Regular oral long-acting morphine with rapid action oromorph for breakthrough pain

Epidural

Intranasal diamorphine

Inhalation of nitrous oxide

Regular oral weak opioid e.g. codeine

A

Achille is a 13-year-old black African boy who has sickle cell disease. He presents with pain in his left leg. He has already taken paracetamol without affect. He has no other medical problems and has no allergies.

219
Q

For each of the children in pain listed below, select the most appropriate next step in their pain management. Each option may be used once, more than once or not at all.

Noah is a 7-year-old boy. He attends the Accident and Emergency department in severe pain. He has been involved in a road traffic accident and has a compound fracture of his femur. He has had several episodes of vomiting. He is extremely agitated. He has no intravenous access.

Distraction

Topical Anaesthetic

Regular oral NSAIDs

IV Morphine via a nurse-controlled pump

IV patient controlled analgesia (PCA) with morphine

Regular oral paracetomol

Regular oral long-acting morphine with rapid action oromorph for breakthrough pain

Epidural

Intranasal diamorphine

Inhalation of nitrous oxide

Regular oral weak opioid e.g. codeine

A

Intranasal diamorphine

220
Q

For each of the children in pain listed below, select the most appropriate next step in their pain management. Each option may be used once, more than once or not at all.

Jake is 3 years old. He has had a hernia repair and has just come back to the ward. He is not allergic to any medicines and other than moderate asthma is fit and well. You are asked to write up some pain relief by the nurses.

Distraction

Topical Anaesthetic

Regular oral NSAIDs

IV Morphine via a nurse-controlled pump

IV patient controlled analgesia (PCA) with morphine

Regular oral paracetomol

Regular oral long-acting morphine with rapid action oromorph for breakthrough pain

Epidural

Intranasal diamorphine

Inhalation of nitrous oxide

Regular oral weak opioid e.g. codeine

A

Regular oral paracetomol

221
Q

For each of the children in pain listed below, select the most appropriate next step in their pain management. Each option may be used once, more than once or not at all.

Charlie is a 13-month-old boy who is attending the community centre to receive his Measles, Mumps & Rubella (MMR) vaccination. He has no medical problems, has no allergies and is not currently on any medication.

Distraction

Topical Anaesthetic

Regular oral NSAIDs

IV Morphine via a nurse-controlled pump

IV patient controlled analgesia (PCA) with morphine

Regular oral paracetomol

Regular oral long-acting morphine with rapid action oromorph for breakthrough pain

Epidural

Intranasal diamorphine

Inhalation of nitrous oxide

Regular oral weak opioid e.g. codeine

A

Distraction

222
Q
# Choose from the options, which best describes the reason for the doctor’s actions in managing Tolla, an 11-year-old girl who has recently arrived in the United Kingdom with her mother. They are originally from Uganda. Tolla has been admitted to the paediatric ward with a chest infection. Her mother is HIV-positive but has not told any of her family. Her husband, Tolla’s father, died of an AIDS-related illness 6 months ago. Tolla has been tested for HIV and her test result has come back positive. Her mother does not wish her to be told, as she is still very upset about her father dying.
Each option may be used once, more than once or not at all.

The hospital manager has asked you to not start antiretroviral therapy as the family do not have legal status to stay in the UK and cannot afford the medication. You start the therapy against this advice.

Non-maleficence

Beneficence

Justice

Truth-telling

Duty

Utility

Rights

Autonomy

A

Duty

223
Q
# Choose from the options, which best describes the reason for the doctor’s actions in managing Tolla, an 11-year-old girl who has recently arrived in the United Kingdom with her mother. They are originally from Uganda. Tolla has been admitted to the paediatric ward with a chest infection. Her mother is HIV-positive but has not told any of her family. Her husband, Tolla’s father, died of an AIDS-related illness 6 months ago. Tolla has been tested for HIV and her test result has come back positive. Her mother does not wish her to be told, as she is still very upset about her father dying.
Each option may be used once, more than once or not at all.

As Tolla’s doctor you feel she should not be told the diagnosis as this will potentially expose her and her family to more harm due to the stigma attached to having HIV. The knowledge that she is HIV-positive may also affect her self-esteem.

Non-maleficence

Beneficence

Justice

Truth-telling

Duty

Utility

Rights

Autonomy

A

Non-maleficence

224
Q
# Choose from the options, which best describes the reason for the doctor’s actions in managing Tolla, an 11-year-old girl who has recently arrived in the United Kingdom with her mother. They are originally from Uganda. Tolla has been admitted to the paediatric ward with a chest infection. Her mother is HIV-positive but has not told any of her family. Her husband, Tolla’s father, died of an AIDS-related illness 6 months ago. Tolla has been tested for HIV and her test result has come back positive. Her mother does not wish her to be told, as she is still very upset about her father dying.
Each option may be used once, more than once or not at all.

Her mother has asked you to tell her the HIV test is negative and as the doctor you do not feel this is ethically right.

Non-maleficence

Beneficence

Justice

Truth-telling

Duty

Utility

Rights

Autonomy

A
225
Q
# Choose from the options, which best describes the reason for the doctor’s actions in managing Tolla, an 11-year-old girl who has recently arrived in the United Kingdom with her mother. They are originally from Uganda. Tolla has been admitted to the paediatric ward with a chest infection. Her mother is HIV-positive but has not told any of her family. Her husband, Tolla’s father, died of an AIDS-related illness 6 months ago. Tolla has been tested for HIV and her test result has come back positive. Her mother does not wish her to be told, as she is still very upset about her father dying.
Each option may be used once, more than once or not at all.

As Tolla’s doctor you feel she has a right to be involved in her own treatment and therefore should be told she is HIV-positive.

Non-maleficence

Beneficence

Justice

Truth-telling

Duty

Utility

Rights

Autonomy

A

Autonomy

226
Q
# Choose from the options, which best describes the reason for the doctor’s actions in managing Tolla, an 11-year-old girl who has recently arrived in the United Kingdom with her mother. They are originally from Uganda. Tolla has been admitted to the paediatric ward with a chest infection. Her mother is HIV-positive but has not told any of her family. Her husband, Tolla’s father, died of an AIDS-related illness 6 months ago. Tolla has been tested for HIV and her test result has come back positive. Her mother does not wish her to be told, as she is still very upset about her father dying.
Each option may be used once, more than once or not at all.

As Tolla’s doctor you feel she should be told she is HIV-positive, as you will be able to offer her more support and coping strategies if she knows more about her diagnosis.

Non-maleficence

Beneficence

Justice

Truth-telling

Duty

Utility

Rights

Autonomy

A

Beneficence

227
Q

The paediatric team is resuscitating a 3-month-old boy who is in Pulseless Electrical Activity (PEA). He was discovered to be blue and lifeless when his parents went to wake him in the morning. The airway has been secured and despite ventilation the child remains blue. An intra-osseous needle has been inserted and two boluses of normal saline have been given. The nurse reports that the temperature of the child is 35°C. You listen to the chest and can hear bilateral breath sounds.

Where is the most appropriate position on the chest to do cardiac compressions?

Between the sternal notch and the nipple line

At the nipple line

Between the nipple line and xiphisternum

None of the above

A

Between the nipple line and xiphisternum

In an infant the heart is lower in relation to the external landmarks than in older children or adults. The area of compression over the sternum should be one fingerbreadth below an imaginary line between the nipples

228
Q

You are in the acute assessment unit and see David, a 15-month-old boy who has a fever of 38.5°C. He has had a runny nose, cough and a fever for 3 days. Since this morning he has slept and has been difficult to wake. His heart rate is raised. He has a purpuric rash with lesions of variable size scattered over his legs which does not disappear with pressure.

Which of the following is the most likely diagnosis?

Non-accidental injury
Henoch–Schönlein purpura (HSP)
Idiopathic thrombocytopenia (ITP)
Acute lymphoblastic leukaemia (ALL)
Septicaemia

A

Septicaemia

He has a purpuric rash, with lesions of variable size. In a febrile child, meningococcal septicaemia is most likely. This may be accompanied by meningitis

229
Q

A 3-year-old boy who is unconscious arrives in Accident and Emergency. You manage his Airway, Breathing and Circulation. His blood glucose is normal. On examination you note that he has bilateral pinpoint pupils. His temperature and other vital signs are otherwise normal.

What is the most likely cause?

Third nerve lesion
Severe hypoxia
Hypothermia
Tentorial herniation
Opiate poisoning

A

Opiate poisoning

Bilateral, pin-point pupils with coma can be caused by a pontine lesion or opiate poisoning. Opiate poisoning may occur in homes with illicit substance abusers or adults on methadone.

230
Q

You are called to see a 3-year-old boy with a high fever. The nurse is worried that he is very sleepy. As you walk into the resuscitation room he makes no spontaneous response. You try calling his name but he makes no response. On stimulation, his eyes open, he moans and he raises his hand and pushes your hand away.

What is this child’s Glasgow Coma Score?

8
9
10
11
12

A

9

231
Q

Ryan, aged 10 months, is rushed to the Accident and Emergency department after being found submerged in the bath. His mother runs screaming into the department saying ‘help my baby, please’.

Which is the next most appropriate step?

Commence bag and mask ventilation
In managing the airway, his head should be in the neutral position.
Commence chest compressions in a ratio of 15:2.
Assess patient, call for help
Remove wet clothing/towels

A
232
Q

Mohammed, aged 8 months, has been vomiting and off his feeds for two days. Initially he had episodes of crying uncontrollably, drawing his legs up into his abdomen as if in pain, and appeared fractious. His mother gave him some oral rehydration solution, but his vomiting continued and he has become lethargic. On admission to hospital he is in shock.

What is the most likely diagnosis?

Gastroenteritis
Malrotation
Strangulated hernia
Intussusception
Meckel diverticulum

A

Intussusception is the most likely cause of the pain and shock. Although this could be a strangulated hernia, this should be evident on clinical examination.

233
Q

Mohammed, aged 8 months, has been vomiting and off his feeds for two days. Initially he had episodes of crying uncontrollably, drawing his legs up into his abdomen as if in pain, and appeared fractious. His mother gave him some oral rehydration solution, but his vomiting continued and he has become lethargic. On admission to hospital he is in shock. Intussusception is suspected.

Mohammed is 8 months old and weighs 8 kg. He needs a bolus of normal saline 0.9% to treat his shock.

What volume of fluid would you give initially?

40 ml
160 ml
320 ml
680 ml
800 ml

A

160ml

This is 20 ml/kg initially, repeated as necessary

234
Q

Mohammed, aged 8 months, has been vomiting and off his feeds for two days. Initially he had episodes of crying uncontrollably, drawing his legs up into his abdomen as if in pain, and appeared fractious. His mother gave him some oral rehydration solution, but his vomiting continued and he has become lethargic. On admission to hospital he is in shock. Intussusception is suspected.

Mohammed has received the fluid bolus of normal saline 0.9% which has improved his condition. From his presentation you suspect he is 10% dehydrated. You receive his laboratory results which reveal a plasma sodium of 138 mmol/L (within the normal range). His continuing fluid loss from vomiting is small and can be ignored. (The maintenance intravenous fluid requirement for a child of this age is 100 ml/kg/24 h.)

What is Mohammed’s total fluid requirement for the initial 24 hours? He weighs 8 kg.

160 ml
320 ml
800 ml
880 ml
1600 ml

A

1600ml

Mohammed’s fluid requirement is calculated by adding:
• Deficit: 10% of 8 kg = 800 ml
• Maintenance: 100 ml/kg/24 h = 800 ml
• Continuing losses: 0 ml
Total = 1600 ml

235
Q

You are called to the resuscitation room where there is a 6-year-old child who has arrived by ambulance. He is a known to have epilepsy and is on anti-epileptic treatment. The child has been having a generalised seizure for 15 minutes. The ambulance crew gave a dose of buccal midazolam 5 minutes ago. The emergency doctor has maintained the airway and has applied oxygen with a non-rebreathe mask. His capillary refill time is less than 2 seconds and his heart rate 120 beats/minute.

What is the next most appropriate management step?

Administer further anti-convulsant
Gain intravenous access
Check blood glucose level
Check pupils
Check conscious level

A

Check blood glucose level

This is the most appropriate next step as, if the patient is hypoglycaemic, the only treatment to stop the fit would be to administer glucose.

236
Q

Seb, a 2-year-old boy, was at his cousin’s birthday party. His mother noticed that he has suddenly developed a widespread urticarial rash and has also become flushed in the face. His vital signs are normal and he has no respiratory compromise.

Which medication would you give?

Intramuscular adrenaline
Oral corticosteroid
Intravenous hydrocortisone
Oral antihistamine
Intramuscular antihistamine

A

Oral antihistamine

In children, the most common causes of acute food allergy are ingestion or contact with nuts, egg, milk or seafood. Urticaria and facial swelling are mild reactions. Immediate management is with an oral antihistamine (e.g. chlorpheniramine) and observed over 2 hours for possible complications.

237
Q

Jenny, a 3-year-old girl, was at a village fete. She suddenly developed swollen cheeks and lips and a widespread urticarial rash. She is rushed to the nearby general practice surgery, where it is noted that her breathing is very noisy. She is distressed and frightened. On auscultation she has widespread wheeze.

Which medication would you give first?

Intramuscular adrenaline
Oral corticosteroid
Intravenous hydrocortisone
Oral antihistamine
Intramuscular antihistamine

A

IM Adrenaline (1:1000)

This child has anaphylaxis, which is life-threatening as she has upper airway obstruction and bronchoconstriction. Priority is to manage the airway and give oxygen via a non-rebreathe mask. The first medication to give would be intramuscular adrenaline.

238
Q

There has been a dramatic decline in the incidence of sudden infant death syndrome in the UK.

Which of the following is the single most important factor responsible for this decline?

Feet to foot of cot
Supine sleeping
Keeping baby in parent’s room until 6 months
Keeping room cool
Parents should not smoke in same room as infant

A

Supine sleeping

All the answers have helped reduce the risk of cot death, but the single most important factor is putting babies to sleep on their backs

239
Q

For each of the following patients seen in the Accident and Emergency department select the most appropriate next step in the management plan. Each option may be used once, more than once, or not at all.

Mohammed, aged 2 months, is found by his mother to be pale and floppy in his cot. The paramedics are giving bag and valve mask ventilation when he arrives in the resuscitation room. His heart rate is 40 beats/minute.

Airway opening manouvres

Bag and mask ventilation

High-flow oxygen

IV access

IV fluids

Check blood glucose

Check pupils

Check conscious level (AVPU)

Secondary survey

Commence cardiac compressions using hands encircling method

Commence cardiac compressions using one hand on the sternum

Commence cardiac compressions using both hands

A

Commence cardiac compressions using hands encircling method

240
Q

For each of the following patients seen in the Accident and Emergency department select the most appropriate next step in the management plan. Each option may be used once, more than once, or not at all.

Nathaniel, a 4-year-old boy, is brought to hospital with shortness of breath. He is able to talk but has oxygen saturation of 90%. His capillary refill time is less than 2 seconds.

Airway opening manouvres

Bag and mask ventilation

High-flow oxygen

IV access

IV fluids

Check blood glucose

Check pupils

Check conscious level (AVPU)

Secondary survey

Commence cardiac compressions using hands encircling method

Commence cardiac compressions using one hand on the sternum

Commence cardiac compressions using both hands

A
241
Q

For each of the following patients seen in the Accident and Emergency department select the most appropriate next step in the management plan. Each option may be used once, more than once, or not at all.

Aisha, a 3-year-old girl, is bought to hospital by the paramedics as she has had a seizure. She is receiving high-flow oxygen, her breathing is regular, and the cardiac monitor shows a heart rate of 100 beats/minute. She is unresponsive to painful stimuli, as she does not flinch when her blood glucose is tested.

Airway opening manouvres

Bag and mask ventilation

High-flow oxygen

IV access

IV fluids

Check blood glucose

Check pupils

Check conscious level (AVPU)

Secondary survey

Commence cardiac compressions using hands encircling method

Commence cardiac compressions using one hand on the sternum

Commence cardiac compressions using both hands

A

Check pupils

242
Q

For each of the following patients seen in the Accident and Emergency department select the most appropriate next step in the management plan. Each option may be used once, more than once, or not at all.

Daniel has been playing football at his friend’s house. He has been brought to Accident and Emergency as he has become confused and is sweaty. He walks into the department.

Airway opening manouvres

Bag and mask ventilation

High-flow oxygen

IV access

IV fluids

Check blood glucose

Check pupils

Check conscious level (AVPU)

Secondary survey

Commence cardiac compressions using hands encircling method

Commence cardiac compressions using one hand on the sternum

Commence cardiac compressions using both hands

A

Check blood glucose

243
Q

For each of the following patients seen in the Accident and Emergency department select the most appropriate next step in the management plan. Each option may be used once, more than once, or not at all.

Kelsey, a 2-year-old girl, is found unconscious in the garden. When she is bought into the resuscitation room she is gasping and moaning.

Airway opening manouvres

Bag and mask ventilation

High-flow oxygen

IV access

IV fluids

Check blood glucose

Check pupils

Check conscious level (AVPU)

Secondary survey

Commence cardiac compressions using hands encircling method

Commence cardiac compressions using one hand on the sternum

Commence cardiac compressions using both hands

A

Airway-opening manouvres